Vous êtes sur la page 1sur 74

© 2010 SPMM Course www.spmmpsychiatrycourse.

com

CONTENTS

Developmental neuroscience
Neuroanatomy
Neuropathology
Neurohistology
Neurochemistry
Neuroendocrinology
Neurophysiology
Neuroimaging
Neuropsychology
Behavioural & Clinical Neurology

MRCPsych Paper 2 Revision


Neurosciences
Answers & Lecture Notes
May 2010

Neurosciences Answers 77
© 2010 SPMM Course www.spmmpsychiatrycourse.com

Developmental Neuroscience:
The nervous system appears between 2.5-4 weeks of gestation in human embryo.

During embryo development the midline neural tube differentiates into the following vesicles
1. Prosencephalon  differentiates into the
a. Telencephalon (cerebrum, striatum and pallidum) and
b. Diencephalon (thalamus, subthalamus, hypothalamus and epithalamus)
2. Mesencephalon (midbrain)
a. Tectum - corpora quadrigemina, made up of the superior and inferior
colliculi
b. Tegmentum containing the red nucleus and periaqueductal grey matter.
3. Rhombencephalon  differentiates into
a. Metencephalon - pons, rostral medulla oblongata and cerebellum.
b. Myelencephalon – caudal medulla oblongata.

Organisation of cortex:
i. Primary areas:
1. Motor cortex – Precentral gyrus; consists of motor, remoter, supplementary motor
and frontal eye fields.
2. Sensory cortex – except olfaction, all other sensations are relayed via thalamus to
sensory cortex. It occupies postcentral gyrus.
ii. Secondary areas:
1. This refers to association cortices that surround primary areas. These areas act as
processors of information and integrate various modalities of data delivered to brain.
iii. Tertiary areas:
1. Also called as heteromodal association cortices, these are located in inferior parietal
lobe and prefrontal and superior temporal areas. These serve the highest order of
processing and may contribute to top-down processes. Prefrontal cortex is thought to
be the last region to mature in human brain.

Homunculus:
 Both motor and sensory systems encode a map of body space in brain corresponding to motor
and sensory representations individually. These are called, motor (precentral gyrus) and
sensory (post central gyrus) homunculus respectively.
 Penfield and Rasmussen‘s pioneering work of focal stimulation during neurosurgery was
pivotal in determining homunculi.
 The spatial representation in brain is disproportionate to actual body measures. For example
thumb occupies a large part of motor homunculus, lip occupies a major part of sensory
homunculus, and thighs receive comparatively smaller area.
 Cortical regions with homunculi have high reorganisation capacity – for example, after an
amputation, the limb area gets substantially reorganised. It is possible that this reorganisation
plays important role in anomalous experiences like phantom limb syndromes.
 This plasticity is also noted if a stroke destroys parts of the somatosensory cortex; remapping
of homunculus to adjacent areas can occur, to preserve normal function. Homunculi are also
noted in cerebellum, supplementary motor area and thalamus.

Neurosciences Answers 78
© 2010 SPMM Course www.spmmpsychiatrycourse.com

Synaptic pruning:

At birth, cellular elements are still developing and migrating in the cortex. By midchildhood,
more neurons and cellular processes are established than in adult years. The developmental task of
childhood years from an anatomic point of view is to prune and to select the most useful (perhaps
the most used) neurons, synapses, and dendrites to preserve for the adult brain. This process of
pruning continues through the early teen years. The synapses that are most important to survival
and optimal function flourish whereas useless connections vanish.

Neuronal numbers can be studied using a wide variety of markers including the density of D2
receptors. Before 5 years of age, D2 receptor density is greater than adult levels, but regresses
during the second decade. Dopamine receptors continue to decrease in adult years, but at a
considerably slower rate of 2.2% reduction per decade. This rate is faster in males than in females.

In schizophrenia, the rate of D2 receptor loss is faster than in healthy comparison subjects: a 1.9%
loss per decade in comparison men and a 6.0% loss per decade in men with schizophrenia.

While excessive pruning is associated with schizophrenia, relative under-pruning is implicated in


autism, where regional size of certain brain components may be more than in controls.

Seeman, P. Pruning during development. Am J Psychiatry 156:168, February 1999

Neural crest cells are first recognized at the lateral margin of the neural placode shortly after gastrulation,
although they are not committed to their diverse fates until later. After dorsal closure of the neural tube,
neural crest cells separate and migrate throughout the embryo to form many structures of ectodermal origin
(eg, dorsal root and autonomic ganglia, peripheral nerve sheaths) and some of mesodermal origin (eg, blood
vessels, melanocytes, adipose tissue, membranous bone, connective tissue, most of the ocular globe).
Terminal differentiation occurs after migration is complete. Three regions of the neural tube generate neural
crest: rhombencephalon, mesencephalon, and prosencephalon, each with a different migratory pattern. The
traditional view of neurocutaneous syndromes is that these syndromes are somehow related to skin and
brain because both are ectodermal derivatives – but in fact, the diverse features result from abnormal neural
crest differentiation into various cell types.

Neurosciences Answers 79
© 2010 SPMM Course www.spmmpsychiatrycourse.com

Neuroanatomy:
Hemispheric lateralisation:
 Most fundamental brain functions are represented bilaterally. Higher levels of associative
functions usually lateralise to one or other hemisphere. For example, language
comprehension is localized to the left temporal cortex, while prosody (tonal modulation of
speech) seems limited to right hemisphere.
 The hemisphere contralateral to the dominant hand is the dominant hemisphere and it
mediates language and speech functions. In right-handed people, the left hemisphere is
dominant.
 Dominance can be tested using Annette‘s handedness scale or Edinburgh handedness
inventory. But handedness is not always same as dominance.
 In 10% of right-handed people the right hemisphere is dominant. In left handed people only
about 20% are right hemisphere dominant as expected, with 64% left hemisphere dominant
and 16% showing bilateral dominance.
 fMRI, PET or functional transcranial Doppler sonography (fTCD) can be used to test
dominance e.g. when a language processing task is undertaken. Invasive methods include
using Wada test (injecting sodium amytal via carotid artery on one side to test if language
gets affected) or using unilateral ECT and testing naming function within few minutes of
recovery. This is essential before undertaking major neurosurgical procedures.
 Dichotic listening is also used to assesses cerebral dominance. Individuals usually
understand speech better with right ear as fibers cross to left hemisphere which is dominant
for speech. When two words are presented simultaneously - one to each ear – preferential
processing takes place for right sided presentation in left lobe dominance.
 The planum temporale is a triangular region on the upper surface of the temporal lobe. It
isimportant for language processing and shows a left-right asymmetry of size in most
brains. In 65% of all individuals the left planum temporale appears to be more developed,
while the right planum temporale is more developed in only 10%. In some people‘s brains,
the planum temporale is more than five times larger on the left than on the right, making it
the most asymmetrical structure in the brain. This asymmetry may be lost or reversed in
schizophrenia.
Left Hemisphere Right Hemisphere
Aphasia Visuospatial deficits
Right-left disorientation Impaired visual perception (agnosias)
Finger agnosia Neglect
Dysgraphia (aphasic) Dysgraphia (spatial, neglect)
Dyscalculia (number alexia) Dyscalculia (spatial)
Limb apraxia Constructional apraxia (Gestalt)
Face recognition (bilateral) Dressing apraxia
Anosognosia
Face recognition (bilateral)
Dragovic, M., Allet, L. & Janca, A. Electroconvulsive therapy and determination of cerebral
dominance. Ann Gen Hosp Psychiatry. 2004; 3: 14.

Frontal structure:
There are four regions of the PFC that are mentioned frequently in the scientific literature. These
are orbitofrontal, dorsolateral prefrontal, anterior cingulate cortex and ventromedial cortex (the
latter two together form the medial frontal surface).
Lewis DA. Structure of the human prefrontal cortex. Am J Psychiatry. 2004; 161[8]: 1366

FRONTAL LESIONS

Neurosciences Answers 80
© 2010 SPMM Course www.spmmpsychiatrycourse.com

I. Effects of unilateral frontal disease, either left or right


A. Contralateral spastic hemiplegia
B. Slight elevation of mood, increased talkativeness, tendency to joke inappropriately
(Witzelsucht), lack of tact, difficulty in adaptation, loss of initiative
C. If entirely prefrontal, no hemiplegia; grasp and suck reflexes or instinctive grasping may be
released
D. Anosmia with involvement of orbital parts
II. Effects of right frontal disease
A. Left hemiplegia
B. Changes as in I.B, C, and D
III. Effects of left frontal disease
A. Right hemiplegia
B. Motor speech disorder with agraphia, with or without oro-buccal apraxia
C. Sympathetic apraxia of left hand
D. Loss of verbal associative fluency; perseveration
E. Changes as in I.B, C, and D
IV. Effects of bifrontal disease
A. Bilateral hemiplegia
B. Spastic bulbar (pseudobulbar) palsy
C. If prefrontal, abulia or akinetic mutism, lack of ability to sustain attention and solve complex
problems, rigidity of thinking, bland affect, social ineptitude, behavioural disinhibition, inability
to anticipate, labile mood, and varying combinations of grasping, sucking, obligate imitative
movements, utilization behavior.
D. Decomposition of gait and sphincter incontinence

An epileptic fit of incessant ‗laughter‘, not necessarily euphoria, is called gelastic seizure. This
occurs with left prefrontal seizures.

Prefrontal syndromes:
Orbital (orbitofrontal) Disinhibited Poor impulse control
(pseudopsychopathic) Explosive outburst, Inappropriate behavior
Dorsolateral (dorsal Disorganized Cognitive dysfunction
convexity) (dysexecutive) Diminished judgment, planning, and insight
Concrete and inflexible
Decreased spontaneous behaviours
Medial (ventromedial) Apathetic Paucity of spontaneous behaviours
(pseudodepressive) Sparse verbal output, reduced social knowledge
and judgement of harmful intent
Higgins, E S.& George, MS. Neuroscience of Clinical Psychiatry, The: The Pathophysiology of
Behavior and Mental Illness, 1st Edition. Lippincott Williams & Wilkins 2007. Page 16

TEMPORAL LESIONS:

Auras of Temporal lobe epilepsy TLE: (MRCPsych favourite)


Autonomic sensations are the most common of auras, causing epigastric aura, salivation,
sometimes vertigo etc.
Forced thinking The individual has a compulsion to think on a certain restricted topic.
Evocation of thought: Intrusion of stereotyped words or thoughts.

Neurosciences Answers 81
© 2010 SPMM Course www.spmmpsychiatrycourse.com

Sudden obstruction to thought flow similar to schizophrenic thought block is also reported.
Panoramic memory: Here the individual recalls expansive memories in incredible detail, as if
running a video show of past.
Psychic seizures: Isolated auras with hallucinations, depersonalisations, micropsia or macropsia,
déjà vu or jamais vu (especially if right sided origin) can occur.
Uncinate crises: Hallucinations of taste and smell of uncinate origin associated with dream like
reminiscence and altered consciousness.
Transient dysphasia in an aura point to left hemisphere origin. Speech automatisms are usually
right sided.
Strong affective experiences are reported – fear and anxiety being very common. Dostoevsky’s
epilepsy refers to ecstatic content in epileptic aura.

Effects of unilateral disease of the dominant temporal lobe


A. Homonymous upper quadrantanopia
B. Wernicke‘s aphasia (word-deafness—auditory verbal agnosia)
C. Amusia (some types)
D. Impairment in tests of verbal material presented through the auditory sense
E. Dysnomia
F. Visual agnosia
G. Occasionally amnesic (Korsakoff) syndrome
II. Effects of unilateral disease of the nondominant temporal lobe
A. Homonymous upper quadrantanopia
B. Inability to judge spatial relationships in some cases
C. Impairment in tests of visually presented nonverbal material
D. Agnosia for sounds and some qualities of music
III. Effects of disease of either temporal lobe
A. Auditory, visual, olfactory, and gustatory hallucinations
B. Dreamy states with uncinate seizures
C. Emotional and behavioural changes
D. Disturbances of time perception
IV. Effects of bilateral disease
A. Korsakoff amnesic defect (hippocampal formations)
B. Apathy and placidity
C. Hypermetamorphopsia (compulsion to attend to all visual stimuli), hyperorality,
hypersexuality, blunted emotional reactivity (Kluver-Bucy syndrome; the full
syndrome is rarely seen in man)

Geschwind syndrome: This is an uncommon type of personality change reported in epilepsy


patients (esp. TLE). It consists of hypergraphia, circumstantiality, interpersonal viscosity,
hyperreligiosity, and hyposexuality. It is thought to be result of lost connectivity among cerebral
areas.
(Commonest personality in epilepsy is characterised by increased suspiciousness and paranoia)

PARIETAL LESIONS
I. Effects of unilateral disease of the parietal lobe, right or left
A. Corticosensory syndrome and sensory extinction
B. Mild hemiparesis
C. Homonymous hemianopia or inferior quadrantanopia (incongruent or congruent) or visual
inattention

Neurosciences Answers 82
© 2010 SPMM Course www.spmmpsychiatrycourse.com

D. Neglect of the opposite side of external space (far more prominent with lesions of the right
parietal lobe—see below)
II. Effects of unilateral disease of the dominant (left) parietal lobe (in right-handed and most
left-handed patients)—additional phenomena include
A. Disorders of language (especially alexia)
B. Gerstmann syndrome (dysgraphia, dyscalculia, finger agnosia, right-left confusion)
C. Tactile agnosia (bimanual astereognosis)
D. Bilateral ideomotor and ideational apraxia
III. Effects of unilateral disease of the nondominant (right) parietal lobe
A. Visuospatial disorders
B. Topographic memory loss
C. Anosognosia, dressing and constructional apraxias (these disorders may occur with lesions of
either hemisphere but one observed more frequently and are of greater severity with lesions of the
nondominant one)
IV. Effects of bilateral disease of the parietal lobes
A. Visual spatial imperception, spatial disorientation, and complete or partial Balint syndrome
described below

OCCIPITAL LESIONS:

I. Effects of unilateral disease, either right or left


A. Contralateral (congruent) homonymous hemianopia, which may be central (splitting the
macula) or peripheral; also homonymous hemiachromatopsia
B. Elementary (unformed) hallucinations—usually due to irritative lesions
II. Effects of left occipital disease
A. Right homonymous hemianopia
B. If deep white matter or splenium of corpus callosum is involved, alexia and color-naming
defect
C. Visual object agnosia
III. Effects of right occipital disease
A. Left homonymous hemianopia
B. With more extensive lesions, visual illusions (metamorphopsias) and hallucinations (more
frequent with right sided than left-sided lesions)
C. Loss of topographic memory and visual orientation
IV. Bilateral occipital disease
A. Cortical blindness (pupils reactive)
B. Anton syndrome (visual anosognosia, denial of cortical blindness)
C. Loss of perception of color (achromatopsia)
D. Prosopagnosia (temporo-occipital), simultanagnosia (parieto-occipital)
E. Balint syndrome (parieto-occipital)

Limbic circuit:

 The limbic system was delineated by James Papez in 1937.


 Paul MacLean in the mid-1950s popularized the concept by linking the structure to emotional
functions
 The Papez circuit consists of the hippocampus, the fornix, the mamillary bodies, the anterior
nucleus of the thalamus, and the cingulate gyrus

Neurosciences Answers 83
© 2010 SPMM Course www.spmmpsychiatrycourse.com

 The boundaries of the limbic system were subsequently expanded to include the amygdala,
septum, basal forebrain, nucleus accumbens, and orbitofrontal cortex.
 Hippocampal formation (Subiculum) → fornix → mammillary bodies → mammillothalamic
tract → anterior thalamic nucleus → genu of the internal capsule → cingulate gyrus →
Parahippocampal gyrus → entorhinal cortex → perforant pathway → hippocampus
 Functions:
 Mediating emotional response (amygdalar connections)
 Influencing the endocrine system and the autonomic nervous system via
hypothalamus
 Hippocampus is involved in recalling previously learned material
 Reward system regulation via nucleus accumberns
 Motivational circuitry (cingulate and DLPFC)

Basal Ganglia:
♣ The basal ganglia is a group of gray matter nuclei in subcortical area. It forms the largest
subcortical structure in the brain
♣ Its components are the caudate nucleus, the putamen and the globus pallidus Putamen and
globus pallidus together forms lenticular/lentiform nucleus.
♣ Caudate and putamen are often clubbed together by the name striatum.
♣ The caudate nucleus is bounded on one side by the lateral ventricle and is divided into a head,
body and tail.
♣ The subthalamic nuclei and the substantia nigra are both functionally related to the basal
ganglia, but are not considered to be part of that structure.
♣ Substantia nigra appears black because of melanin pigment.
♣ The globus pallidus is divided into an external and an internal segment. The substantia nigra
is divided into a pars compacta and a pars reticulata.
♣ Important afferents are from glutamatergic corticostriate projection
♣ Another important tract is dopaminergic nigrostriatal projection
♣ Five important circuits are described ( Alexander)
o Motor circuit
o Oculomotor circuit
o Dorsolateral prefrontal circuit (executive)
o Anterior cingulate circuit (motivation)
o Lateral orbitofrontal circuit (social intelligence)
♣ Functions:
o The basal ganglia are involved in the planning and programming of movement
o They also have a role in the processes by which an abstract thought is converted into
voluntary action
o The basal ganglia also play a role in some cognitive processes, particularly the
caudate nucleus.
o Lesions of the caudate disrupt performance on tests involving object reversal and
delayed alternation.
OCD Both reduced and increased volumes of caudate nuclei reported.
Patients have higher caudate blood flow. Increased caudate
metabolism has been found to reduce after effective treatment of
the OCD.
Tourette’s syndrome Striatal dopaminergic dysfunction
Huntington chorea degeneration of the striatum (mainly caudate nucleus) &
selective loss of GABAergic neurons
Wilson disease Copper deposits in the lenticular nuclei (pallidus and putamen)

Neurosciences Answers 84
© 2010 SPMM Course www.spmmpsychiatrycourse.com

CO poisoning Acute bilateral anoxic damage of basal ganglia


Hemiballismus Subthalamic nucleus damage (especially infarction)
Parkinsonism Substantia Nigra depigmented; Lewy bodies seen. Striatal
overactivity may lead to bradykinesia
Fahr's disease Progressive calcium deposition in the basal ganglia. (early onset
- schizophreniform psychoses and catatonic symptoms; later
onset – dementia and choreoathetosis)

Hypothalamic structures:
 Ventromedial hypothalamus acts as satiety centre; lateral hypothalamus as feeding centre.
 Animals with lesion of ventromedial hypothalamus – hyperphagia, obesity noted.

Corpus callosum:
This is the largest bundle of fibres that connect the two cerebral hemispheres; the other such
bundles are anterior commissure, posterior commissure, hippocampal commissure and habenular
commissure. The pericallosal artery/arteria pericallosa derived from the anterior cerebral artery
provides blood supply of the corpus callosum.

Cerebellum
 Cerebellum has the important role of preparing a motor plan and predicting balance needed
between muscle groups to carry out the intended action smoothly.
 Cerebellar lesions produce ataxia and coarse intentional tremors, along with hypotonia, past
pointing and pendular knee jerk. Even imagined mental activities activate cerebellum.
 Cognitive dysmetria: Cognitive dysmetria is a difficulty in coordinating and monitoring the
process of receiving, processing, and expressing information thought to be a result for
disrupted cortico-cerebellar circuitry in schizophrenia.

Inferior Olivary Nucleus


 Inferior olivary nucleus serves motor coordination via projecting climbing fibers to the
cerebellum.
 Axons arising from cells in the inferior olive cross and enter the inferior cerebellar
peduncle to reach the contralateral cerebellar cortex.
 Inferior olivary lesions lead to appendicular ataxia that can be tested using finger-nose
test. This results in motor incoordination of the contralateral arm and leg.
 (NB Cerebellar lesions result in ipsilateral motor incoordination)
Ruigrok TJ. Cerebellar nuclei: the olivary connection. Prog Brain Res. 1997;114:167-92.

Blood supply to brain:


 The internal carotid artery enters the circle of Willis and divides to form the anterior
cerebral and middle cerebral arteries.
 The anterior cerebral artery supplies the medical and superior lateral aspects of the
cerebral cortex to the parietal/occipital border.
 The middle cerebral artery supplies the greater part of the lateral aspect of the cerebral
cortex. The middle cerebral artery supplies the Broca‘s and Wernicke‘s area in the
dominant hemispheres.
 The posterior cerebral artery arises from basilar artery and supplies the inferomedial
temporal lobe and the occipital lobe.
 Internal capsule is supplied by the circle of Willis

Neurosciences Answers 85
© 2010 SPMM Course www.spmmpsychiatrycourse.com

 Medulla is supplied by posterior inferior cerebellar arteries and anterior spinal artery –
branches of vertebral arteries.
 Pons is supplied by the basilar artery that runs along the midline of the pons.

Anterior circulation Posterior circulation


Carotid system TIA Vertebrobasilar system TIA
Amaurosis fugax Diplopia, vertigo, vomiting
Aphasia Choking and dysarthria
Hemiparesis Ataxia
Hemisensory loss Alexia without agraphia
Hemianopic visual loss Hemisensory loss
Hemianopic visual loss
Transient global amnesia
Tetraparesis
Loss of consciousness (rare)

 The lateral medullary syndrome or posterior inferior cerebellar artery (PICA)


thrombosis, or Wallenberg's syndrome, is a brain-stem infarction presenting as acute
vertigo with cerebellar signs. It follows thrombo-embolism in the PICA or its branches,
vertebral artery thrombo-embolism or dissection.

Ipsilateral Contralateral
Facial numbness (Vth) Spinothalamic sensory loss
Diplopia (VIth) Hemiparesis (mild, unusual)
Nystagmus
Ataxia (cerebellar)
Horner's syndrome
IXth and Xth nerve lesions

Cerebrovascular disease & lobar deficits

Artery Supply Lesion effects


Anterior cerebral artery medial surface of the brain, Bilateral anterior cerebral
including the ventromedial artery infarct
frontal lobe, the cingulum, = quadriparesis (legs worse
the premotor cortex, and the than arms) and akinetic
motor strip mutism (ventromedial or
cingulate syndrome)
Recurrent artery of Huebner head of the caudate nucleus agitated confusional state
(branch of ACA) that with time evolves to
akinesia, abulia, and mutism,
along with personality
changes
anterior branches of the lateral prefrontal cortex Planning deficits, impairment
upper division of the middle of working memories, and
apathy. (DLPFC dysfunction)

Neurosciences Answers 86
© 2010 SPMM Course www.spmmpsychiatrycourse.com

cerebral artery
Anterior communicating basal forebrain Akinesia and personality
artery changes (orbitofrontal) and
striking confabulatory
amnesia that resembles
Wernicke-Korsakoff
syndrome.

http://emedicine.medscape.com/article/1135866-overview

CSF flow:
► CSF is secreted within the choroid plexus in the lateral, third and fourth ventricles and at
a rate of 300 ml/day, which is almost protein free.
► Route:
o From lateral ventricle to 3rd ventricle via interventricular foramina of Monroe
o From 3rd to 4th Ventricle via cerebral aqueduct of sylvius
o From 4th ventricle to subarachnoid space via Foramen of Magendie (middle) and
Foramen of Luschka (two, lateral)
► The body of the lateral ventricle lies immediately below the corpus callosum and they are
separated by septum pellucidum.
► The third ventricle lies between thalamus and hypothalamus
► The fourth ventricle lies above the pons and just below the cerebellum
► Obstruction to its circulation commonly occurs within third or fourth ventricle, leading to
non-communicating hydrocephalus. Obstruction to CSF flow in the subarachnoid space
leads to communicating or normal pressure hydrocephalus.

Neuropathology:
Alzheimer’s dementia:
Gross changes:
 Diffuse atrophy
 Flattened cortical sulci and
 Enlarged cerebral ventricles.

Histological changes:
 Neuronal loss (particularly in the cortex and the hippocampus).
 Synaptic loss and
 Granulovascular degeneration of the neurons: small vacuoles with central granules, in the
cytoplasm of neurons especially in the temporal lobes
 Senile plaques
 Amyloids are fibrils of multimeric chains of peptides deposited extracellularly.
 They have a beta pleated sheet confirmation.
 The peptide involved is called Aß (beta A4) peptide.

Neurosciences Answers 87
© 2010 SPMM Course www.spmmpsychiatrycourse.com

 Aß is cleaved from a larger transmembrane protein—amyloid-ß precursor


protein—by the action of ß- and -secretases, and its formation is prevented by the
action of -secretase.
 Amyloid is insoluble.
Plaques vary in appearance, and two main subtypes are recognised.
Diffuse plaques:
o They consist largely of non-fibrillar extracellular Aß.
o They are not related to the degree of cognitive decline
o Diffuse plaques contain the same peptides as those responsible for amyloid
formation in the neuritic plaques. However, these peptides are not
polymerized to form fibrils and lack beta sheet configuration
o Only neuritic plaques are counted in neuropathological tests.
Neuritic plaques:
o They contain Aß in the form of amyloid fibrils, among which are
irregularly swollen dystrophic neurites (degenerated neuronal processes).
o The neurites are well visualised with silver stains; they may be seen as an
eosinophilic mass on haematoxylin & eosin stains.
o Neuritic plaques may contain a dense central core of amyloid.
o Microglia and astrocyte processes are present towards the periphery of
neuritic plaques.
o Seen in Down syndrome and, to some extent, in normal aging
o Amyloid sensitive stain Congo red, under polarized light, demonstrates the
"apple green" birefringence of the stained object, due to beta pleat sheets.

 Neurofibrillary tangles
 Composed of cytoskeletal elements, primarily abnormally phosphorylated tau
protein
 Tau is a peptide required for microtubule assembly. Microtubules are essential to
transport of materials down axons.
 Beta A4 peptide interacts with cholinergic receptors and this interaction
stimulates the abnormal phosphorylation of tau. The hyperphosphorylated tau is a
major constituent of the tangle. It is also present in the degenerated neurites.
Hence both tangles and neuritic plaques can be identified by staining with
antibody to the abnormal tau.
 Not unique to Alzheimer's disease
 Also occur in Down syndrome, dementia pugilistica (punch-drunk syndrome),
Parkinson-dementia complex of Guam, Hallervorden-Spatz disease, and the
normal elderly.
 Tangles need not always indicate dementia e.g. Down‘s
 Tangles first appear in the hippocampus
 Most tangles are faintly basophilic. They can be impregnated with silver or
immunostained for tau to facilitate their light microscopic detection.
 Tangles are formed within the neuronal cell body and most remain intraneuronal.
 However, when neurons degenerate, the tangles persist extracellularly, although
they lose their basophilia.
 The earliest pattern of involvement is usually not associated with clinical disease:
tangles and neuropil threads are restricted to parts of the entorhinal cortex and the
CA1 field of the hippocampus.
 As dementia develops, tangles and neuropil threads accumulate in increasing
density in other parts of the hippocampus and medial temporal neocortex, and

Neurosciences Answers 88
© 2010 SPMM Course www.spmmpsychiatrycourse.com

then in other cortical regions and in subcortical grey matter structures such as the
hypothalamus and thalamus.
 A staging scheme devised by Braak and Braak (1995) is widely used to describe the
extent of tangle related abnormalities (distribution from entorhinal cortex to isocortex) in
AD and correlates well with the severity of dementia. Stages V-VI operationally define
AD.
 The number and distribution of tangles increases as cognitive decline increases.
 When both neuritic plaques and tangles are present, the presence of even a few tangles in
a single field in the neocortex suggests significant cognitive decline
 There is also an association between the numbers of neuritic plaques and the degree of
cognitive decline. However, this is less easy to show than the relationship between
tangles and cognitive decline.
 Best neuropathological correlate of decline is the number of synapses. The marker for
synapses has been antibody to synaptophysin, a protein found in the presynaptic
endings.

 Cerebral amyloid angiopathy (CAA):


 This is the accumulation of Aß in the walls of blood vessels (particularly arteries
and arterioles) in the cerebral cortex and overlying leptomeninges.
 This affects about 30% of normal elderly people but over 90% of patients with AD,
in whom the angiopathy tends also to be much more severe.
 CAA is an important cause of strokes in the elderly. Most of these are
haemorrhagic; CAA is confined to superficial cerebral blood vessels, rupture of the
amyloid laden blood vessels usually causes relatively superficial, lobar
haemorrhages that may extend into the subarachnoid space.

 Hirano bodies
 Rod shaped eosinophilic bodies in the cytoplasm of neurons; may be set free in the
extracellular space if the neuron dies.
 Hirano bodies are intracellular aggregates of actin and actin-associated proteins
 They are frequently seen in hippocampal pyramidal cells
 Other abnormalities:
 reduction in the density of synaptic proteins in the cerebral cortex
 neuronal loss:
o Specific cellular pattern of neuronal loss is noted in the subiculum of the
hippocampal formation and layers II and IV of the entorhinal cortex.
o The affected cells are precisely those that interconnect the hippocampal
formation with the association cortices, basal forebrain, thalamus, and
hypothalamus, structures crucial to memory.
o This pattern of neuronal loss isolates the hippocampal formation from its
input and output and probably contributes to the memory disorder in
Alzheimer patients
 astrocytic gliosis
 microglial activation.

Vascular dementia
 Primarily affects small- and medium-sized cerebral vessels
 Multiple parenchymal lesions spread over wide areas of the brain
 Binswanger's disease, (subcortical arteriosclerotic encephalopathy) is characterized by
the presence of many small infarctions of the white matter that spare the cortical regions

Neurosciences Answers 89
© 2010 SPMM Course www.spmmpsychiatrycourse.com

Lewy Body dementia:


 Lewy bodies are weakly eosinophilic, spherical, cytoplasmic inclusions.
 In Parkinson‘s disease they are confined to substantia nigra; in DLB they are also present in
many areas of the cerebrum including the temporal lobe, the cingulate gyrus and the frontal
lobes. They may also be found in the dorsal motor nucleus of the vagus.
 Cortical lewy bodies are less conspicuous, less eosinophilic and lack clear halo compared to
those in substantia nigra. Cortical Lewy bodies appear as regions of homogeneous
eosinophilic staining in the neuronal cytoplasm, usually associated with eccentric
displacement of the nucleus.
 There is no simple correlation between number of Lewy bodies and cognitive decline.
 Antibody to protease ubiquitin can be used to identify Lewy bodies. Staining with alpha
synuclein antibodies is an excellent tool for detecting both Lewy bodies.
 Lewy bodies--in Parkinson's disease and DLB contain accumulations of alpha synuclein .
 Alpha synuclein accelerates reuptake of dopamine in neurons and this dopamine overload
might be toxic.
 A high proportion of patients with DLB/PDD (about three quarters) also have AD-type
neuropathological abnormalities.Here the plaque/tangle burden associated with dementia is
less than that required by pure Alzheimer's disease.
 Lewy neurites—these are nerve cell processes that contain aggregates of -synuclein. These
abnormal structures can occur in both DLB/dementia of Parkinson‘s disease and idiopathic
Parkinson‘s disease and are most numerous in the CA2/3 region of the hippocampus and in
the substantia nigra.
 Some patients with DLB show microvacuolation of the cerebral cortex, predominantly in the
medial temporal regions. This can lead to misdiagnosis of prion disease.

Love, S. Neuropathological investigation of dementia: a guide for neurologists Journal of Neurology,


Neurosurgery, and Psychiatry 2005;76(Supplement 5 ):v8-v14;

Fronto temporal dementia:


It is associated with three types of underlying histology:
1. frontal lobe degeneration type:
a. Most common type
b. spongiform degeneration or microvacuolation of the superficial neuropil seen
chiefly from layers III and V
c. loss of large cortical nerve cells
d. Gliosis is minimal
2. Pick‘s type:
a. Pick's disease is characterized by a preponderance of atrophy in the
frontotemporal regions.
b. These regions also have Loss of large cortical nerve cells, abundant gliosis, and
neuronal Pick's bodies, which are masses of cytoskeletal elements.
c. Abnormal swollen neuronal cells, with loss of Nissl‘s substance, oval in shape
with peripherally displaced nucleus are called Pick cells
d. Pick's bodies are seen in some postmortem specimens but are not necessary for
the diagnosis. These are argentophilic, tau and ubiquitin reactive filamentous
inclusions.
e. Hirano bodies may also be seen albeit with a lesser frequency than in
Alzheimer‘s.

Neurosciences Answers 90
© 2010 SPMM Course www.spmmpsychiatrycourse.com

3. Motor neurone disease type.


a. Cerebral atrophy is less marked; limbic areas are largely preserved
b. Loss of large cortical nerve cells, microvacuolation, and mild gliosis.
c. Ubiquitinated but not tau-immunoreactive inclusions are present within the
frontal cortex and hippocampus
d. MND pathology in anterior horn cells seen.
Neary, D & Snowden, J. Fronto-temporal Dementia: Nosology, Neuropsychology, and
Neuropathology. Brain & Cognition 1996;31:176-87

Tauopathies: (diseases with tau deposits)


Alzheimer‘s
Pick‘s
Progressive supranuclear palsy
Corticobasal degenerations
Frontotemporal dementia with Parkinsonism (FTDP-17)
Note: chromosome 17 plays a role in tau processing
Synucleinopathies: (alpha synuclein deposits)
Parkinson‘s
DLB
Multisystem atrophy

Crutzfeld Jacob disease:


 Three forms exist: sporadic (most common), familial and variant CJD (vCJD - related to
bovine spongiform encephalopathy).
 There are no characteristic gross pathologic features of CJD because of the typical short
course of the disease.
 Persons living beyond 6 months to a year may have some degree of generalized cerebral
atrophy.
 Microscopically CJD shows a spongiform encephalopathy secondary to neuropil
vacuolisation.
 Many round to oval vacuoles are seen in the neuropil of cortical gray matter - vacuoles
may be single or multiloculated. The vacuoles may coalesce to microcysts.
 Most cases of CJD also demonstrate neuronal loss and gliosis.
 Prion protein (PrPc) is a normal neuronal cell surface encoded by a gene on chromosome
20. This is converted via a conformational change to an abnormal form designated as
PrPSc. This abnormal form is protease-resistant and can accumulate in the central nervous
system of affected persons. This accumulation triggers further conversion of normal PrPc
to PrPSc and accounts for the degenerative changes in the cerebral cortex.
 These abnormal PrP's can be transmitted from a person with spongiform encephalopathy
to another person via pituitary extracts, corneal transplants, dural grafts, and
contaminated electrodes from neurosurgical procedures.
 The PrP can be identified in tissues with immunoperoxidase staining.
 An abnormal protein called 14-3-3 can be found in the CSF by immunoassay, but this
protein is non specific and may be found in association with viral encephalitis and stroke.
It is less frequent in variant CJD.
 In familial cases of CJD, the typical EEG changes are often lacking, and the 14-3-3
protein is absent from CSF half the time.
 The presence of particular polymorphisms at codon 129 of PrP may have an influence on
susceptibility to disease. The amino acids methionine (M) or valine (V) may be present.

Neurosciences Answers 91
© 2010 SPMM Course www.spmmpsychiatrycourse.com

In 37% of healthy persons, both inherited PrP genes code for methionine, and half have
M/V. 73% of persons with sporadic CJD have the M/M phenotype, and 100% of persons
with variant CJD have this phenotype.
 The MRI is the most useful supportive diagnostic test in variant CJD. A characteristic
abnormality seen in the posterior thalamic region (pulvinar sign) is highly sensitive and
specific for variant CJD.
 The pulvinar sign has been found in more than 90% of pathologically proven vCJD cases.
FLAIR sequences of MRI are most likely to show the abnormality.

Huntington’s dementia:
 Pathologically there is severe loss of small neurons in the caudate and putamen with
subsequent astrocytosis.
 Characteristic protein deposits form nuclear inclusions in neurons of HD patients.
 With the loss of cells, the head of the caudate becomes shrunken and there is "ex vacuo"
dilatation of the anterior horns of the lateral ventricles.

Neurosciences Answers 92
© 2010 SPMM Course www.spmmpsychiatrycourse.com

Schizophrenia:
Gross changes:
♣ A decrease in brain weight, brain length and volume of the cerebral hemispheres
enlargement of the lateral ventricles
♣ Decreased thalamic volume
♣ Reduced tissue volume in temporolimbic structures including hippocampus, amygdala,
parahippocampal gyrus,
♣ Left temporal horn enlargement
♣ White-matter reductions in parahippocampal gyrus or hippocampus, and
♣ An increased incidence of a cavum septi pellucidi is noted.
♣ Pallidal volume reduction is noted especially in preneuroleptic era, in the catatonic
subgroup. Enlargement of basal ganglia can be seen in schizophrenia as a consequence of
treatment with classical neuroleptics, which can be reversed by the use of atypical
substances.
♣ There is some evidence from postmortem studies for disturbed cerebral asymmetry in
schizophrenia, for example the planum temporale.
♣ Schizophrenia-like psychosis is commoner in temporal lobe epilepsy when the focus is in
the left hemisphere.
♣ The planum temporale, the posterior superior surface of the superior temporal gyrus, is a
highly lateralized brain structure involved with language. In schizophrenic patients a
consistent reversal of the normal left-larger-than- right asymmetry of planum temporale
surface area is noted. Heschl's gyrus (primary auditory cortex) showed no differences
between the left and right sides.

Histological changes:
 No evidence for astrogliosis in schizophrenia
 Reduced cell numbers or cell size has been described especially affecting neurons in the
hippocampus and DLPFC.
 Increase in neurone density, which may relate to the observed decrease in neurone size
(with decreased dendritic arborization and a decreased neuropil compartment) has been
reported.
 Subtle cytoarchitectural anomalies were described in the hippocampal formation, frontal
cortex, e.g. a significant cellular disarray in the CA3–CA4 interface
 synaptic studies in the hippocampus and DLPFC in schizophrenia show decrements in
presynaptic markers
 these changes may reflect a reduction in the number of synaptic contacts formed and
received in these areas,
 This supports hypotheses of excessive synaptic pruning in schizophrenia.
 glutamatergic synapses may be especially vulnerable in the hippocampus and perhaps the
DLPFC, with predominantly GABAergic involvement in the cingulate gyrus.

Harrison PJ. The neuropathology of schizophrenia. A critical review of the data and their
interpretation. Brain 1999; 122: 593–624

Alcohol related brain damage:


 Wernicke's encephalopathy is characterized by degenerative changes including gliosis
and small hemorrhages in structures surrounding the third ventricle and aqueduct (i.e. the

Neurosciences Answers 93
© 2010 SPMM Course www.spmmpsychiatrycourse.com

mamillary bodies, hypothalamus, mediodorsal thalamic nucleus, colliculi, and midbrain


tegmentum), as well as cerebellar atrophy.
 Brain shrinkage can be found in uncomplicated alcoholism, which can largely be
accounted for by loss of white matter. Some of this damage appears to be reversible.
 Alcohol-related neuronal loss has been documented in specific regions of the cerebral
cortex (superior frontal association cortex), hypothalamus (supraoptic and paraventricular
nuclei), and cerebellum.

Mood disorders:
 A strong association between mood disorder and the number and severity of focal signal
hyperintensities on T2-weighted images has been established.

These white matter hyperintensities (WMH) occur particularly in the deep subcortical
white matter and to a lesser extent in the basal ganglia and periventricularly.
 They are seen in excess in bipolar and unipolar mood disorder, with an odds ratio of
3 to 7.
 In major depression, WMH are particularly common in elderly subjects, where they are
linked to risk factors for, and the presence of, vascular disease. This finding is consistent
with a robust epidemiological association between the two conditions
 WMH confer a poor prognosis in major depression and bipolar disorder.

 Lithium treatment increases cortical grey matter volume suggesting that lithium is
neurotrophic. Lithium may also enhance neurogenesis and inhibit apoptosis
 Antidepressants may affect neuronal morphology - regenerate monoaminergic axons promote
neurogenesis and prevent the loss of dendritic spines seen in some animals.
 Antipsychotics alter synaptic and neuronal morphology, particularly in the caudate–putamen
and may increase glial density in the prefrontal cortex.

Harrison PJ. The neuropathology of primary mood disorder. Brain 2002;125:1428–49.

Autism:
 Hypoplasia of cerebellar vermis and to some extent the cerebellar hemispheres is
documented.
 Purkinje cell count in cerebellum is significantly lower.
 Inconsistent changes noted in neocortex. Some suggest increased cortical volume, probably
elated to reduced pruning.

HIV neuroscience:
 The major HIV-1 receptors are CD4 and CD8; various chemokine receptors e.g.
 CXCR4 and CCR5 are considered as HIV-1 co-receptors.
 CD4+ helper T lymphocytes are the major routes of multiplication and entry, apart from
monocytes. Infected CD4+ T cells and monocytes, which circulate in the blood, are the
potential source of CNS infection.
 The strains of HIV which are isolated from the brain have the characteristic of infecting
macrophages more than lymphocytes. Macrophage-tropism is related to a mutation in a
specific region of gp120, the external glycoprotein of the virus. In the late stages of the
infection, active replication of the virus generates more of these mutants and the
compromised immune system permits the escape of these mutants, leading to
predominance of macrophage-trophic strains.

Neurosciences Answers 94
© 2010 SPMM Course www.spmmpsychiatrycourse.com

 In order to enter the brain, HIV-1 must cross the BBB using mechanisms that remain
unclear. The generally accepted model is the "Trojan Horse hypothesis". HIV enters the
CNS as a passenger in cells trafficking to the brain via CD4 T cells or monocytes.
 Virus accumulation in perivascular regions has been demonstrated as a proof for the above
model. The mechanisms of endothelial cells infection remain a controversial issue – as
CD4 expression is doubtful in these cells.
 An alternative hypothesis of HIV-1 neuro-invasion proposes the entry of free HIV-1 by
migration between or, transcytosis of endothelial cells.
 Theoretically all the main cell types of the CNS, astrocytes, oligodendrocytes, neurons,
perivascular macrophage and microglia, can be infected by HIV-1 since they possess the
receptors and/or co-receptors for HIV-1 entry, but only the latter two are the most
commonly infected cells by HIV-1. Most studies have indicated an absence of in vivo
infection in neurons - It is unclear whether detection of infected neurons is complicated by
the loss of the infected neuronal populations.
Mechanism of neuropathogenesis:
 Two components of this mechanism are:
 The direct effect of the HIV-1 infection
 The indirect consequence of infection comprising the secretion of cytokines and
neurotoxins.

 The infected macrophages and microglia participate actively in the neurodegeneration by:
1) shedding viral proteins and 2) releasing significant amount of cytokines and neurotoxins
into the CNS. 3) Tat and TNF-α contribute to the disruption of the blood brain barrier,
which in turn become more permeable to infected monocytes and cytokines present in the
periphery.

 The secreted pro-inflammatory cytokines activate microglia and astrocytes which in turn
secrete neurotoxins. In addition, the alteration of astrocyte function results in an increase in
the level of neurotoxicity in the brain.

 Neuronal injury via apoptosis is currently believed to be produced by toxic products


released directly by HIV-infected macrophages and microglia or by activated astrocytes.
Some of these factors have been identified: they include the platelet activating factor,
quinolinic acid, nitric oxide, and some metabolites of arachidonic acid, which are
neurotoxic, and tumour necrosis factor, which is toxic for oligodendrocytes and can cause
demyelination.

The HIV neuropathology:


1. Infiltration of macrophages into the CNS;
2. Formation of microglial nodules;
3. Multinucleated giant cells from virus-induced fusion of microglia and/or
macrophages in central white and deep gray matter;
4. astrocyte activation and damage;
5. Neuronal loss particularly in hippocampus, basal ganglia and caudate nucleus.
6. A variable degree of white matter pathology with myelin damage
7. Accumulation of lipid macrophages in extreme cases, with
8. The presence of HIV-1 in the cerebral spinal fluid (CSF).

The current method used to predict stage of disease, to monitor disease progression, and to
formulate treatment strategies is to determine viral load (actual number of viral particles found in

Neurosciences Answers 95
© 2010 SPMM Course www.spmmpsychiatrycourse.com

a cubic millimeter of blood). Viral load is the preferred measure for monitoring disease
progression, treatment responsiveness, and treatment resistance. CSF viral load is not established
as an accurate indicator of CNS disease related to HIV.

Most common presentation is HIV related dementia, followed by depression. Psychosis is seen
only in 10% of HIV infected individuals.

Neurohistology:
 The human brain contains approximately 1011 neurons (nerve cells) and approximately
1012 glial cells.
 Brodmann‘s 47 areas are divided according to cytoarchitecture of brain.

Neocortex organisation:

o The neocortex is made up of six layers, with pial surface above layer 1 to the white
matter below layer 6.
o The pyramidal neurons with their triangular-shaped cell bodies make up nearly 75% of
the cortical neurons. Stellate cells (25%) are present in all the layers except layer 1.

LAYER NAME PREDOMINANT CELLS


1 Molecular/agranular Glial cells, dendrites from neurones of deeper
layers and the horizontal cells of Cajal.
2 External granular layer Granule cells
3 Internal granular layer Granule cells
4 Internal pyramidal layer Large pyramidal cells
(inputs)
5 External pyramidal layer Large pyramidal cells.
(outputs)
6 Multiform layer Mixture of all cells, spindle cells.

The cerebellar cortex is three layered.


The molecular layer consisting of basket cell and stellate cell,
Purkinje layer consisting of Purkinje cell and
Granular layer consisting of granule and Golgi cell.

Blood brain barrier:


 The concept of the blood brain barrier was first introduced by Paul Ehrlich. He found that
intravenous injection of dyes into the bloodstream stained all the tissues in most organs
except the brain.

Neurosciences Answers 96
© 2010 SPMM Course www.spmmpsychiatrycourse.com

 The blood- brain barrier is located in endothelial cells of capillaries of the brain.
 These brain endothelial cells are different to those found in peripheral tissues in various
ways:
 Brain endothelial cells are joined by tight junctions of high electrical resistance
providing an effective barrier against molecules.
 In peripheral endothelial cells there is good transcellular movement of molecules.
There is no such movement in brain endothelial cells.
 Brain capillaries are in contact with foot processes of astrocytes which essentially
separate the capillaries from the neurones.
 Lipid soluble molecules, such as ethanol and caffeine are able to penetrate through the barrier
relatively easily via the lipid membranes of the cells.
 In contrast, water soluble molecules such as sodium and potassium ions are unable to
transverse the barrier without the use of specialized carrier- mediated transport mechanisms.
 Inflammation such as meningitis weakens the blood brain barrier
 There are some areas of the brain that do not have a blood- brain barrier. These are so called
circum-ventricular organs e.g. subfornical organ, area postrema (chemo receptor trigger
zone), median eminence and posterior pituitary. Here the capillaries are fenestrated like those
in peripheral tissues.

Glial cells
 These are cells with supportive metabolic functions; they also participate in
neuromodulation e.g. via neurosteroids.
 There are three types:
o Astrocytes: Most numerous of all three. These are star shaped cells which enable
nutrition of neurons, breakdown of some neurotransmitters, and maintaining the
blood-brain barrier.
o Oligodendrocytes are seen in CNS (not in peripheral nerves, where Schwann
cells replace them). They produce myelin sheaths which help in saltatory
conduction (pole to pole jumping), which quicken the process of signal
transmission.
o The microglia, are descendants of macrophages. They are scavenger cells which
clear neuronal debris following cell death.
o Ependymal cells are a special type of glia that cover the ventricles and enable
CSF circulation.
Special neuronal cell types:
Purkinje cells are a class of GABAergic neurons located in the cerebellar cortex only. Purkinje
cells send inhibitory projections to the deep cerebellar nuclei, and constitute the sole output of all
motor coordination in the cerebellar cortex.
Granule cells are found within the granular layer of the cerebellum, layer 4 of cerebral cortex,
the dentate gyrus of the hippocampus, and in the olfactory bulb.
Stellate cells are found in layer IV of cerebral cortex (from thalamus feeding forward to
pyramidal cells) and also in cerebellum.
Large pyramidal cells called Betz cells are seen in primary motor cortex.Betz cells are pyramidal
cell neurons located within the fifth layer of the grey matter in the primary motor cortex. These
neurons are the largest in the central nervous system, sometimes reaching 100 μm in diameter.
Betz cells represent about 10% of the total pyramidal cell population in layer V of the human
primary motor cortex.

Migration:
 Glial-guided neuronal migration in the cerebral cortex occupies much of the first 6
months of gestation.

Neurosciences Answers 97
© 2010 SPMM Course www.spmmpsychiatrycourse.com

 Neuronal migration errors result in neurons failing to reach the cortex and instead reside
in ectopic positions. A group of such incorrectly placed neurons is called a heterotopia.
 Thalamic axons that project to the cortex initially synapse on a transient layer of neurons
called the subplate neurons. In normal development, the axons subsequently detach from
the subplate neurons and proceed superficially to synapse on the true cortical cells. The
subplate neurons then degenerate.
 Some, not every brain from persons with schizophrenia reveals an abnormal persistence
of subplate neurons, suggesting a failure to complete axonal pathfinding in the brains of
these persons.
 Synaptogenesis occurs very rapidly from the second trimester through the first 10 years
or so of life. The peak of synaptogenesis occurs within the first 2 postnatal years.
 Myelination begins prenatally; it is largely complete in early childhood, but does not
reach its full extent until late in the third decade of life.
 It is now known that continuous neurogenesis takes place in certain brain regions
(particularly the dentate gyrus of the hippocampus) in adults.

Neurochemistry:

3 types of synapses:
Chemical synapse:
Presynaptic neuron releases chemical molecule on stimulation. This molecule serves to propagate
the impulse further via the next neuron or acts on the next neuron to bring on a molecular effect.
Electrical synapses:
They bring on the response by electrical communication without chemical exchange
Conjoint synapses:
These have both electro-chemical properties.

Neurotransmitters:
Chemical substances that are synthesised in the presynaptic neuron and released to synaptic cleft
to transmit impulses e.g. acetylcholine
Neuromodulators:
Similar to neurotransmitters but instead of impulse relay, these substances modify the response of
the receptor to the neurotransmitter e.g.
Neurohormones:
These are released by nerve cells into the systemic circulation rather than into the synapse e.g.
prolactin
Most of these above mentioned substances can coexist in various combinations within a single
neuronal terminal.

Synaptic transmission:
Presynaptic neuron synthesises, transports and stores the chemical messenger. Synthesis generally
takes place in cell body / soma which contains the essential protein synthesis machinery. From
here axonal transport occurs and the chemical reaches the synaptic terminal. The storage before
eventual release is within the synaptic vesicle. Membrane fusion and exocytosis leads to release.

Upon release the neurotransmitter occupies receptors present on the surface of postsynaptic
neuron. Receptors can be broadly classified into metabotropic (those that induce

Neurosciences Answers 98
© 2010 SPMM Course www.spmmpsychiatrycourse.com

metabolic/synthetic changes upon stimulation – G protein coupled) and ionotropic (those that
result in ion flow predominant in a single direction).

Receptors have finite number and are saturable if there is an over secretion of neurotransmitter.
Neurotransmitters exhibit specificity in receptor interaction. One neurotransmitter can have more
than one receptor types; but within a given receptor site only a particular chemical conformation
can be accommodated (lock and key). Receptor binding is often competitive; relative synaptic
concentrations of competing molecules decide the receptor activity. Most receptors are bound
reversibly i.e. following dissociation of the neurotransmitter, the receptor falls back to its
physiological status quo. Some molecules can act irreversibly producing structural alterations in
the protein of receptor complexes.

Classification of receptors:

Receptors may be categorized into four general categories: (1) ligand-gated channels
(ionotropic), in which binding of a chemical messenger alters the probability of opening
of transmembrane pores or channels; (2) those in which the receptor proteins are coupled
to intracellular G proteins as transducing elements (metabotropic); (3) those consisting of
single membrane-spanning protein units that have intrinsic enzyme activity (for example,
having tyrosine kinase activity); and (4) those termed ligand-dependent regulators of
nuclear transcription (including receptors for corticosteroids such as testosterone).

Ionotropic receptors result in quick response (GABAA benzodiazepine); G protein coupling


(metabotropic) is comparatively slower process (most antipsychotics, antidepressants).

Ion channel receptors


► Class 1
► Fast – millisecond activity
► Made up of four or five protein subunits making up a pore like structure.
► Inhibitory neurotransmitter action leads to entry of Cl- while excitatory action results in
entry of Ca2+ or other cations.
► e.g. GABAA, NMDA, 5HT3 subtype
The GABA-A receptor's structure is typical of most ligand-gated (ionotropic) receptors [‗doughnut with a
hole in the centre‘ or ‗rosette‘ shaped]. It is made up of five protein subunits arranged in a circle to form
a pore, or channel, that remains closed until GABA binds to the recognition site. Each protein subunit is
actually a string of amino acids which passes in and out of the cell membrane four times. At the
extracellular end of this string is a large N-terminal; this end-chain is thought to mediate GABA-channel
interactions. In the middle of the string is a large intracellular loop of amino acids with four sites where
phosphorylation occurs
G-protein-coupled receptors
► Class 2
► Slow receptors
► Trans-membrane protein: all the heterotrimeric G protein-coupled receptors that
have been characterized to date are proteins that span the cell membrane seven
times (serpentine receptors).
► Cyclise mediated second messenger activation (GTP, ATP etc)
► Gs-proteins are stimulatory; Gi-proteins are inhibitory to the adenylate cyclase
► Third variant acts via phospholipase C
► Influence protein synthesis eventually – longer lasting effects.
► e.g. DA receptors, most 5HT except 3, NEN and neuropeptides including opioid
receptors are G coupled.

Neurosciences Answers 99
© 2010 SPMM Course www.spmmpsychiatrycourse.com

Cessation of neurotransmitter action is via


1. reuptake back to presynaptic neuron via special transporters (very relevant to
psychopharmacology)
2. enzymatic breakdown at the cleft
3. removed by glia or plasma circulation
Feedback control of a neurotransmitter may exist at various points
1. presynaptic synthesis
2. release
3. reuptake regulation
4. autoreceptor mediated presynaptic inhibition
5. independent postsynaptic inhibition via a different neuronal network

Glucocorticoid receptors:
These are part of a superfamily of receptors that have in common a cysteine-rich DNA-binding
domain; a ligand-binding domain at or near the carboxyl terminal of the receptor; and a relatively
variable amino terminal region. When a ligand binds to one of them, it becomes a transcription
factor and binds to DNA via zinc fingers. Other receptors in the family include the receptors for
progesterone, androgen, and 1,25-dihydroxycholecalciferol (Vitamin D). Many receptors of this
family are orphan receptors, for which the ligands are still unidentified
The glucocorticoid receptor is located mainly in the cytoplasm but migrates to the
nucleus as soon as it binds its ligand. In contrast, the estrogen and the triiodothyronine
(T3) receptors bind hormones directly in the nucleus itself.

Neurotransmitters:

Monoamines Amino acids Peptides


Dopamine GABA endorphins
Norepinephrine Glycine cholecystokinin
Epinephrine Glutamate angiotensin II
Serotonin neurotensin
Acetylcholine corticotropin-releasing
Histamine hormone

Dopamine:
Source: tyrosine l-dopa  dopamine
Rate limiting step: tyrosine hydroxylase
Breakdown enzymes: Monoamine oxidase (MAO) & Catechol-o-methyl transferase (COMT).
There are two types of MAO - MAO-A more selectively metabolizes norepinephrine and
serotonin and MAO-B more selectively metabolizes dopamine.
Breakdown product: Homovanillic acid
Reuptake: dopamine transporter (cocaine inhibits this)
Function: motivation, novelty seeking, reward circuitry (addictions), arousal and motor
movement gating in basal ganglia
Pathways:
Long: Nigrostriatal (movement), mesocortical and mesolimbic.
Short: Tuberoinfundibular (prolactin inhibiting), incertohypothalamic
Ultrashort: amacrine celss in retina, olfactory system.

Neurosciences Answers 100


© 2010 SPMM Course www.spmmpsychiatrycourse.com

Disorders: Low in Parkinson‘s; high in positive psychotic symptoms at mesolimbic area; may be
low in anhedonia and negative symptoms in mesocortical area.
Receptors: 5 types; D1 - 5
All G protein coupled
D1-like  D1 & D 5; increase adenylate cyclase (stimulatory)
D1 exclusively postsynaptic; resistant to antagonism
D5 more limbic in distribution; 10times higher dopamine affinity
D2-like  D2,3& 4 ; decrease adenylate cyclase (inhibitory)
D4 is found primarily in the frontal cortex and clozapine has a high affinity.
D4-selective antagonists do not have antipsychotic efficacy.

Clinical relevance: Kapur proposed that in the normal individual, the role of mesolimbic
dopamine is to attach significance or ‗salience‘ to an external stimulus, or an internal thought,.
This converts a neutral piece of information into an attention grabbing information (Kapur, 2003).
In acute psychosis where hyperdopaminergic state is noted in mesolimbic system, insignificant
events and perceptions receive inappropriate salience. For example, an innocuous smile of a
stranger may be given high degree of ‗aberrant salience‘ leading to delusional elaborations. On a
similar note when such aberrant salience is attached to internally generated self-speech,
hallucinations may be experienced. Antipsychotics are claimed to "dampen the salience" of these
abnormal experiences - do not erase the symptoms - but provide the platform for a process of
psychological resolution.
Kapur, S. Psychosis as a state of aberrant salience: a framework linking biology, phenomenology, and
pharmacology in schizophrenia. Am J Psychiatry 2003; 160:

Norepinephrine:
Source: tyrosine l-dopa dopamine  norepinephrine  epinephrine
Rate limiting step: tyrosine hydroxylase;
Synthetic enzymes: dopamine--hydroxylase modulates norepinephrine production;
phenylethanolamine-N-methyltransferase modulates conversion of NEN to epinephrine.
Breakdown enzymes: Monoamine oxidase (MAO – A especially) & Catechol-o-methyl
transferase (COMT).
Breakdown product: 3-methoxy-4-hydroxyphenylglycol (MHPG) & VMA – vanillyl mandelic
acid. MHPG is the major metabolite in CNS while VMA is major metabolite from peripheral
nervous system/endocrine system.
Reuptake: noradrenaline reuptake channel (tricyclics, reboxetine inhibit this)
Function: arousal, anxiety, mood regulation, autonomic mediation
Localisation:
Locus coeruleus – extensive projection on to cortex and other brain areas
Disorders: Low in depression and affected in panic/anxiety disorders.
Receptors: 2 major types; α and β.
divided into 1 and 2
1 receptors phospholipase C coupled; mostly postsynaptic
2 receptors Gi coupled ; mostly presynaptic autoreceptors
-receptors Gs coupled; predominate locus ceruleus – may regulate 
1-receptors – high affinity to norepinephrine and 2-receptors – high affinity to epinephrine

Neurosciences Answers 101


© 2010 SPMM Course www.spmmpsychiatrycourse.com

Clinical relevance: Chronic antidepressant treatment induces a reduction in β adrenoreceptor


density around 2 weeks after starting antidepressants; this correlates with therapeutic effects.
Unmedicated suicide victims show higher density of β adrenoreceptors.

Serotonin:
Source: tryptophan5 hydroxy l-tryptophan  serotonin
Rate limiting step: availability of tryptophan (hence it is possible to conduct tryptophan
depletion studies and manipulate 5HT system)
Synthetic enzymes: tryptophan hydroxylase
Breakdown enzymes: MAO (preferentially MAO-A)
Breakdown product: 5-hydroxyindoleacetic acid (5-HIAA)
Reuptake: noradrenaline reuptake channel (tricyclics, reboxetine inhibit this)
Function: mood, perception of pain, feeding, sleep-wake cycle, motor activity, sexual behaviour,
and temperature regulation.
Localisation:
Most serotonin is in periphery (gut, platelets) but cannot cross BBB.
Median and the dorsal raphe nuclei; widespread cortical projection; also spinal cord
Disorders: low serotonin levels  increased depression, aggression, suicide, and impulsivity;
regulate dopamine system – role in psychosis
Receptors: 14 known subtypes of serotonin receptors (5-HT1A, 5-HT1B, 5-HT1D, 5-HT1E, 5-HT1F,
5-HT2A, 5-HT2B, 5-HT2C, 5-HT3, 5-HT4, 5-HT5A, 5-HT5B, 5-HT6, and 5-HT7)
All except 5-HT3 are G-protein-coupled receptors; 5HT3 predominant in gut; associated with
motility.
5-HT1A receptors – Gi coupled postsynaptic; antidepressant response; sexual behaviour
5-HT1B receptors – Gi coupled presynaptic;
5-HT1D receptors – Gi coupled - both presynaptic and postsynaptic.
5-HT2 receptors - phospholipase C coupled; postsynaptic; antagonism leads to antipsychotic
response (atypicals) and sedation; LSD causes 5-HT2 stimulation; down regulation noted after
antidepressant treatment / ECT.
5-HT6 may be involved in antidepressant action

Clinical relevance: An increased density of 5HT2 binding sits has been shown in post mortem
studies of depressed / suicidal patients. The increase in 5HT2A receptors is most prominent in
dorsolateral prefrontal cortex and in platelets of medication naïve patients. A reduction in 5HT 1A
receptors has also been noted in cortex. In contrast, long term antidepressant treatment has been
shown to reduce 5HT2 receptors and increase 5HT1A function. But these changes may not be
causative in antidepressant action as they predate any clinical response in those who are started
on antidepressant therapy. Of note, ECT treatment actually increases 5HT2 receptors. Most
directly acting 5HT1A agonists have poor antidepressant activity.

Serotonin receptors: Major actions

Receptor Action

Neurosciences Answers 102


© 2010 SPMM Course www.spmmpsychiatrycourse.com

5HT1A Antidepressant (agonist), anxiolytics (partial agonist)

5HT1B Aggression

5HT1D Antimigraine (antagonist)

5HT2A Antipsychotic (antagonist); hallucinogens (agonist / partial agonist);


implicatd in wrking memory; seen in platelets and smooth muscles too

5HT2B Stimulation may produce cardiac valvular fibrosis (dexfenfluramine)

5HT2C Anxiogenic and anorexic effect (agonists)

5HT3 Antiemetic (antagonist)

5HT6 Possible antipsychotic/antidepressant action (antagonism)

5HT7 Regulation of circadian rhythm

DOPA decarboxylase (DDC) is an enzyme implicated in 2 metabolic pathways, synthesizing 2 important


neurotransmitters, dopamine and serotonin (Christenson et al., 1972). Following the hydroxylation of
tyrosine to form L-dihydroxyphenylalanine (L-DOPA), catalyzed by tyrosine hydroxylase, DDC
decarboxylates L-DOPA to form dopamine. This neurotransmitter is found in different areas of the brain
and is particularly abundant in basal ganglia. Dopamine is also produced by DDC in the sympathetic
nervous system and is the precursor of the catecholaminergic hormones, noradrenaline and adrenaline in
the adrenal medulla. In the nervous system, tryptophan hydroxylase produces 5-OH tryptophan, which is
decarboxylated by DDC, giving rise to serotonin. DDC is a homodimeric, pyridoxal phosphate-dependent
enzyme.
http://www.ncbi.nlm.nih.gov/entrez/dispomim.cgi?id=107930

Acetylcholine:
Source: choline and acetyl-coenzyme A
Rate limiting step: availability of choline (and acetylcholine concentration itself)
Synthetic enzymes: choline acetyltransferase
Breakdown enzymes: acetylcholinesterase – rapid metabolism
Breakdown product: choline
Reuptake: no reuptake. Degraded choline is re up-taken and recycled.
Function: Modulate arousal, learning, memory, rapid eye movement sleep,
pain perception, and thirst and parasympathetic mediation.
Localisation: 1. ascending system of cholinergic neurons originating in the
reticular formation
2. cholinergic cells in the nucleus basalis of Meynert
Disorders: reduced cholinergic function in Alzheimer's dementia; dopamine
balance affected in Parkinson‘s

Neurosciences Answers 103


© 2010 SPMM Course www.spmmpsychiatrycourse.com

Receptors: Muscarinic receptors - G-protein-coupled.


Five subtypes (M1, M2, M3, M4, and M5)
Nicotinic receptors - ion channels;
more in peripheral parasympathetic system;
Less common than M receptors in CNS – mediates
attention.

Cortical choline acetyl transferase (ChAT) is reduced to a greater extent (85%) in patients with
hallucinations in Lewy body dementia than in those without hallucinations (50%). This is more
pronounced in parieto-temporal regions, hippocampus and entorhinal cortex. As a consequence,
brain acetylcholine levels are reduced in DLB similar to Alzheimer‘s. This may partially explain
the altered sleep-wake patterns seen in DLB and also the response of hallucinations to
acetylcholinesterase inhibitors.

-Aminobutyric acid (GABA)


Source: glutamic acid (glutamate)
Synthetic enzymes: glutamic acid decarboxylase (GAD)
Rate limiting step: GAD catalysis
Breakdown enzymes: GABA transaminase
Breakdown product: Broken down to glutamate, and then eventually to succinic
acid
Reuptake: reuptake into both presynaptic nerve terminals and
surrounding glial cells; uptake system is bidirectional and
both temperature- and ion-dependent process; (inhibited by
tiagabine)
Function: Mediates anxiety, seizure cessation, and actions of
benzodiazepines, barbiturates, and alcohol.
Localisation:  Do not occur in discrete pathways but widely
distributed throughout the brain especially in
interneurons.
 Most common inhibitory amino acid neurotransmitter
 Seen in 60% of the synapses of brain.
Disorders Role in anxiety disorders and alcoholism; may have a role in
many other disorders including epilepsy and Huntington’s.
Receptors:  GABAA and GABAB
 GABAA – opens chloride channel; inhibitory – leads to
hyperpolarization; made of five subunits and at least 14
subunit subtypes
 GABAB receptor is G-protein-coupled; baclofen is
selective agonist

Glutamate:
Source: 1. from 2-oxoglutarate and aspartate by aspartate
aminotransferase,
2. from glutamine by glutaminase, or
3. from 2-oxoglutarate by ornithine aminotransferase
Regulation: accumulation of precursors such as glutamine or by end-product
inhibition

Neurosciences Answers 104


© 2010 SPMM Course www.spmmpsychiatrycourse.com

Reuptake: Largely glial uptake with conversion to glutamine


Function: Important metabolic role – intermediary in oxidation pathway
(malate shuttle), immediate precursor of all GABA in CNS,
intermediary in ammonia cycle; NMDA - memory acquisition,
developmental plasticity, epilepsy, and ischemic brain injury.
NMDA receptor mediates long-term potentiation

Localisation:  major excitatory neurotransmitter – wide distribution in


neocortical projections
 Hippocampus and cerebral cortex.

Disorders excitotoxic glutamate toxicity in stroke/schizophrenia/seizures


suspected. NMDA antagonists can cause hallucinations – e.g.
PCP, ketamine
Receptors:  metabotropic - 8 in total; 3 groups. Group I - mGluR1&
mGluR5 – linked to phospholipase C
 Ionotropic: NMDA and non-NMDA
 NMDA - made up of subunits with distinct binding sites
for glutamate, glycine, phencyclidine (PCP), magnesium,
and zinc.
 Non NMDA – kainate binding or AMPA type.

Glycine
 Synthesized primarily from serine by serine trans-hydroxymethylase and glycerate
dehydrogenase, both of which are rate limiting.
 Glycine acts as a mandatory adjunctive neurotransmitter for glutamate activity and an
independent inhibitory neurotransmitter at its own receptors.
 The excitatory glycine site on the NMDA receptor is called non-strychnine-sensitive glycine
receptor.
 Strychnine-sensitive glycine receptor is an inhibitory receptor seen in spinal cord. May
reduce negative symptoms of schizophrenia.

Peptides:
 Substance P is the primary neurotransmitter in most primary afferent sensory neurons (Pain)
and is also seen in the striatonigral pathway. Levels of substance P have been reported to be
markedly reduced in patients with Huntington's chorea.
 Neurotensin coexists with dopamine in some axon terminals. Neurotensin-related peptides
may benefit psychotic symptoms. Neurotensin may have reward-enhancing effects through its
antidopaminergic actions in the prefrontal cortex
 Cholecystokinin has been implicated in the pathophysiologies of schizophrenia, eating
disorders and movement disorders. CCK infusion induces panic attacks in people with panic
disorder. CCK antagonists may have anxiolytic effects.
 Somatostatin is also known as growth hormone-inhibiting factor.
 Neuropeptide Y has been shown to stimulate the appetite.
 Endogenous Opioids:
Three major branches of the endogenous opioid peptide family-
Proopiomelanocortin (POMC)-derived peptides e.g. -endorphin (most potent natural
opioids); other products of POMC are ACTH and melanocyte stimulating hormone MSH.

Neurosciences Answers 105


© 2010 SPMM Course www.spmmpsychiatrycourse.com

Proenkephalin-derived peptides - Met-enkephalin and Leu-enkephalin


Prodynorphin-related peptides - four major peptides, called dynorphin A, dynorphin B,
and two neoendorphins,  and .
 Opiate receptors
 receptor = morphine, opiate alkaloids, -endorphin, and enkephalins
 receptor = enkephalins
 receptor = dynorphin
 receptor is different from the other opiate receptors in that naloxone does not reverse
agonist activity. Phencyclidine, haloperidol, and steroids are agonists at the  receptor
and indicate that  receptors play a role in motor function and possibly drug abuse.
 The general role of the endogenous opioids includes the regulation of pain (supraspinal
analgesia; spinal analgesia), anxiety, and memory. Other likely effects include
regulation of sexual activity, feeding, temperature, and blood pressure.

Endocannabinoids:
o Anandamide (weak ligand) and 2-arachnidonylglycerol (strong) are formed from
arachidonic acid and ethanolamine – belong to endocannabinoid family.
o The two types of cannabinoid receptors, central (CB1) and peripheral (CB2), bind
tetrahydrocannabinol (THC), the active ingredient of marijuana.
o Anandamide lowers intraocular pressure, decreases activity level, and relieves pain.

Neurotrophins:
These are substances that act as polypeptide growth factors influencing proliferation,
differentiation and survival of neuronal cells. The best characterised neurotrophic factors are
Nerve growth factor (NGF), brain derived neurotrophic factor (BDNF), neurotrophin 3 and
neurotrophin 4.

According to neurotrophin hypothesis neurons compete with each other during development for
limited resource of growth factors at the target region. Those neurons that are highly

Neurosciences Answers 106


© 2010 SPMM Course www.spmmpsychiatrycourse.com

responsive, e.g. via high affinity binding sites, survive while others undergo programmed cell
death. Incorrect targeting of axons may also lead to apoptosis (programmed cell death).

The yin-yang hypothesis states that proneurotrophins elicit apoptosis, while neurotrophins
promote dendritic spine growth. Similarly, in the adult brain, proneurotrophins elicit long-term
depression, while neurotrophins lead to long-term potentiation.

BDNF may have a role in LTP of memory. In animals, chronic stress leads to down regulation
of BDNF. BDNF has been shown to have trophic effects on serotonergic and noradrenergic
neurons. SSRIs and other antidepressants including ECT up regulate BDNF. The time course
coincides with observed therapeutic actions of these drugs.

A single nucleotide polymorphism in the BDNF gene on chromosome 11p13 results in an


amino-acid substitution of valine (val) with methionine (met) at codon 66 (Val66Met) reducing
BDNF activity. BDNF met/met mice demonstrate increased anxiety. Clinical studies in humans
have demonstrated that subjects with the Val66Met allele have impaired hippocampal
activation and performance. It is controversial if met polymorphism increases the risk of
clinical disorders or not.

Neuroendocrinology:

Neurosciences Answers 107


© 2010 SPMM Course www.spmmpsychiatrycourse.com

Anterior pituitary GH growth hormone

LH– luteinizing hormone ( a gonadotrophin)

FSH – follicle stimulating hormone ( a gonadotrophin)

ACTH – adreno corticotrophic hormone (corticotrophin)

TSH – thyroid stimulating hormone

Posterior pituitary Vasopressin & Oxytocin (release only)

Hypothalamus CRH - corticotrophin releasing hormone

GHRH - growth hormone releasing hormone

GnRH - gonadotrophin releasing hormone

TRH thyrotrophin releasing hormone

Somatostatin

Prolactin inhibitory factor (PIF)

Hypothalamic- anterior Pituitary – Adrenal HPA axis:

*HPA axis is an important topic in the MRCPscyh exams*.

 HPA axis is involved in regulation of stress response.


 A diurnal variation in ACTH and cortisol levels occurs in humans, with peak cortisol levels
occurring around 6:00-7:00 AM.
 The regulatory controls in the levels of cortisol are corticotropin-releasing hormone (CRH)
produced by hypothalamus and adrenocorticotropic hormone (ACTH) produced by anterior
pituitary.
 Amygdalar inputs activate while hippocampal inputs reduce CRH release from
hypothalamus.
 ACTH stimulates the release of cortisol from the adrenal cortex.
 Cortisol thus produced in turn inhibits both CRH and ACTH in a negative feedback loop to
maintain homeostasis.
 In conditions of chronic stress this feedback fails and continuous excess of glucocorticoids is
produced with deleterious consequences.
 The most deleterious effect with respect to brain is seen in the hippocampus as GR
(glucocorticoid receptors) are abundant here. The following are seen;
 Impaired memory performance.
 Disrupted long-term potentiation (LTP).
 Atrophy of hippocampal dendrites.
 Shrinkage of the hippocampus.

Neurosciences Answers 108


© 2010 SPMM Course www.spmmpsychiatrycourse.com

 Decreased neurogenesis.
 This is associated with increased dendritic arborization of neurons in the basolateral
amygdala, (may explain memory bias towards negative events)
 Patients with hypercortisolism (Cushing's syndrome) may have depression, mania, confusion,
and psychotic symptoms.
 Apathy, fatigue, and depression are common symptoms in hypocortisolism (Addison's
disease).
 High cortisol levels with loss of the usual diurnal variation in levels have been reported
mainly in patients with depression (especially melancholic with somatic syndrome), but also
in some patients with mania (especially psychotic), obsessive-compulsive disorder,
schizoaffective disorder, or eating disorders.
 A subgroup of depressed patients has also demonstrated an attenuated ACTH response to
CRH stimulation.
 In PTSD hypocortisolemia is seen in a subgroup of patients; this may be due to aberrant
feedback to the pituitary due to excessive glucocorticoid receptors – probably a genetic
vulnerability.
 Low cortisol is also seen in chronic fatigue and fibromyalgia.
Dexamethasone suppression test:
 1mg dexamethasone is give at 11PM with basal cortisol sampling; later next day at
8AM, 4PM and 11PM samples are obtained again. Any one sample >5mcg/L is classed
as DST non suppression.
 The dexamethasone suppression test (DST) is used to demonstrate failure of feedback
suppression of ACTH/CRH and continuous production of endogenous cortisol despite
administration of exogenous steroid (dexamethasone). Normally administering
dexamethasone must reduce cortisol in plasma. This is because of intact HPA function
leading to reduced ACTH and CRH. In depression and other psychiatric
hypercortisolemic states (also in organic hypercortisolemic states such as Cushing‘s) this
does not occur.
 The sensitivity of the DST (rate of a positive outcome, or nonsuppression of cortisol)
in major depression is modest (about 40%- 50%) but is higher (about 60%-70%) in very
severe, especially psychotic, affective disorders, including major depression with
psychotic as well as melancholic features, mania, and schizoaffective disorder.
 The specificity (true negative outcome) of the DST in normal control subjects is
above 90%, but it varies from less than 70% to more than 90% in psychiatric conditions
that often need to be separated from major affective disorders.
 In addition, a number of major medical conditions, pregnancy, Cushing‘s including
severe weight loss and use of alcohol and certain other drugs (hepatic enzyme inducers
that reduce dexamethasone availability - barbiturates, anticonvulsants, and others) can
produce false positive results.
 False negative results are due to Addison‘s, steroid therapy, hypopituitarism, high
dose benzodiazepines or indometacin.
 Positive initial DST status in major depression does not add significantly to the
likelihood of antidepressant response, and a negative test is not an indication for
withholding antidepressant treatment.
 Some data suggest that DST-positive depressions (cortisol nonsuppression) are less
likely than DST-negative cases (cortisol suppression) to respond to a placebo.
 Failure to convert to normal suppression of cortisol with apparent recovery from
depression suggests an increased risk for relapse into depression, poor prognosis and even
suicidal behaviour.

Neurosciences Answers 109


© 2010 SPMM Course www.spmmpsychiatrycourse.com

The dexamethasone suppression test: an overview of its current status in psychiatry. The APA Task
Force on Laboratory Tests in Psychiatry. Am J Psychiatry 1987; 144:1253-1262

Thyroid Hormones:
► TRH from hypothalamus (thyrotropin releasing hormone) stimulates secretion of TSH –
thyrotropin from pituitary. TSH in turn stimulates thyroid gland to synthesise and release
thyroxine T4 and triiodothyronine T3.
► T4 is the predominant form of thyroid released by the gland, but T3 is the more
biologically potent form. T4 is converted into T3 by the target organs as well as the brain.
► Increased serotonergic transmission concomitant with decreased 5-HT1A sensitivity and
increased 5-HT2A sensitivity is noted with exogenous thyroid
► Nerve growth factor genes are activated by T3 during development but not in the adult
brain. 5-HT1A stimulation with chronic T3 administration altered the production of BDNF
in the hippocampus, although neither did so alone. Hence a synergistic relationship
between 5-HT1A receptors and thyroid hormones in the expression of BDNF is
postulated.
Thyroid disturbances
Physical Symptoms Psychiatric Symptoms
Hyperthyroidism Tachycardia Anxiety
Weight loss Irritability
Heat intolerance Trouble in concentrating
Sweating Emotionally labile
Psychomotor agitation
Hypothyroidism Fatigue Depressed mood
Weight gain Decreased libido
Cold intolerance Psychomotor retardation
Dry skin Poor memory

► Normally injecting TRH produces a brisk increase in concentration of TSH produced. If


this increase does not occur it is called a blunted TRH stimulation test. This correlates
with depression.
► Blunted TRH response overlaps with DST non suppression in 30% patients only; TRH
test may detect a different subpopulation than DST suppression.
► Mania, alcohol withdrawal and anorexia can also cause blunted TRH response.
► Addition of T3 and T4 as supplements to antidepressant treatment has been shown to
accelerate response in some patients, particularly women.
► Lithium produces hypothyroidism.
► Hypothyroidism is implicated in rapid cycling of stable bipolar patients.
Growth Hormone:
 Produced by anterior pituitary. Excess causes acromegaly in adults or gigantism in
children; low levels are associated with dwarfism.
 Somatostatin is the growth hormone-inhibiting factor while a stimulating factor GHRH
is also produced by hypothalamus.
 Exercise, stress increases GH release.
 Sleep associated increase is noted.
 The growth hormone response to GHRH and its normal sleep-associated release pattern
is altered in depression and in anorexia nervosa.
Prolactin:
 Produced by anterior pituitary

Neurosciences Answers 110


© 2010 SPMM Course www.spmmpsychiatrycourse.com

 Prolactin-inhibiting factor (dopamine), prolactin-releasing hormone (possibly linked to


TRH) control prolactin levels.
 Prolactin levels are increased during pregnancy, nursing and during sleep and exercise.
Antipsychotics remove the inhibitory control of dopamine by blocking D2 receptors in
tuberoinfundibular tract. This leads to Hyperprolactinaemia – gynecomastia in males
and galactorrhea in females. Long standing prolactin increase may lead to
osteoporosis.
Posterior Pituitary: Vasopressin & Oxytocin
o Vasopressin (ADH - antidiuretic hormone) and oxytocin are peptides differing from each
other in only two amino acids in their sequences.
o Both are synthesized in the supraoptic nuclei and paraventricular nuclei of the
hypothalamus.
o These nuclei have 2 types of cells.
o Magnocellular neurons –predominantly in supraoptic nucleus – produce oxytocin and
vasopressin that are transported and released from axon terminals in posterior pituitary.
o Parvocellular neurons, in contrast secrete hypothalamic regulatory hormones – that regulate
anterior pituitary hormones. They do not synthesize anterior pituitary hormones per se.
o Vasopressin is thought to play a role in attention, memory, and learning. Release of
vasopressin is increased by pain, stress, exercise, morphine, nicotine, and barbiturates and
is decreased by alcohol.
o SIADH is noted in antidepressant, carbamazepine or antipsychotic use.
o Oxytocin has also been implicated in mammalian bonding behavior, particularly in the
initiation and maintenance of maternal behavior, social bonding, and sexual receptivity.
Pineal gland
 Melatonin regulates circadian rhythms. It has both synchronizing and phase-shifting
properties in the regulation of biological rhythms
 It is synthesized from serotonin in the pineal gland by the action of serotonin-N-acetylase
and 5-hydroxyindole-O-methyltransferase.
 The major regulator of melatonin synthesis is the light-dark cycle, with synthesis increased
during darkness. But fluctuations occur even without light-dark cues, but this cycle is
longer.
 The pineal gland appears to be regulated by a major -adrenergic mechanism, and -
antagonists such as propranolol decrease melatonin synthesis.

Neuroendocrine changes in sleep:


Start of sleep – increased testosterone (helpful perhaps!!)
SWS – GH increases; somatostatin increases; cortisol drops
REM sleep – reduced melatonin
Early morning sleep – prolactin increases.

Neurophysiology:
Resting membrane potential:

Neurosciences Answers 111


© 2010 SPMM Course www.spmmpsychiatrycourse.com

An action potential is initiated in the axon hillock when the synaptic signals received by the
dendrites and soma are sufficient to raise the intracellular potential from -70 mV to the threshold
potential of - 55mV. When this potential is reached, the Na+ channels present in the axon initial
segment will open. This Na+ influx causes rapid reversal of the membrane potential from -90 to
+40 mV. When the membrane potential reaches +40mV, the Na+ channels close and the voltage-
gated K+ channels open. K+ ions move out of the axon, and ―repolarizes‖ the membrane.

Electrophysiology of sleep:
Clinical measures of sleep:
 Actigraphy: This is used to quantify circadian sleep wake patterns and to detect movement
disorders during sleep; it uses a motion sensor.
 Polysomnography: This includes EEG, electromyogram EMG, electrooculogram EOG. ECG,
oximetry and respiratory monitor can be added if needed. It is indicated in diagnosis and
monitoring of sleep apnoea, narcolepsy, restless legs, REM behavioural disorder etc. Some of
the terms used in this are
 Sleep latency: time from lights out to sleep onset.
 REM latency: Time from sleep onset to first REM episode. Normally it is about 90
minutes in adults.
 Non REM latency: Time from sleep onset to first Non REM episode.
 Sleep efficieny: (Total sleep time/total time in bed) X 100.
 Multiple sleep latency test: This is used to assess daytime somnolence and day time REM
onset.

Sleep architecture:
 The average length of sleep is approximately 7.5 hours per night
 Sleep is made up of non-rapid eye movement (NREM) sleep and rapid eye movement (REM)
phases.
 NREM sleep: (75% sleep in adults)
o Most physiological functions are markedly lower than in wakefulness (decreased muscle
tone, respiration, temperature and heart rate).
o Classified as stages 1 to 4 with increasing amplitude and decreasing frequency of EEG
activity.
 Stage 1 sleep (the drowsy period – 5%) - very light; when awoken from this stage
one denies being asleep. Low voltage theta activity, V sharp waves.
 Stage 2 sleep (45% - most common) shows the development of sleep spindles andK
complexes.
 Stage 3 sleep (12% of sleeptime) - <50% delta waves.
 Stage 4 sleep (13% of sleeptime)- >50% delta waves. Physiological functions at the
lowest. 3 & 4 together constitute slow wave sleep (SWS). SWS dominates initial
part of the sleep.
o Features of non-REM sleep
1. Reduced recall of dreaming if awoken
2. Increased Parasympathetic activity
3. Decreased heart rate, systolic blood pressure, respiratory rate, cerebral blood flow
4. abolition of tendon reflexes
5. Upward ocular deviation with few or no movements.
6. sleep terror occurs in NREM – hence the confusion on waking up.
 REM sleep (25% sleep in adults)

Neurosciences Answers 112


© 2010 SPMM Course www.spmmpsychiatrycourse.com

o Characterized by a high level of brain activity and physiological activity similar to those in
wakefulness.
o EEG shows low-voltage, mixed frequency (theta and slow alpha) activity similar to awake
state - Saw-tooth waves are seen.
o The REM episodes increase in length as the night unfolds.
o Darting eye movements but other muscles still paralysed (if not, as in REM sleep behavioural
disorder, violent movements may occur)
o Autonomic functions similar to awake state – penile erection noted.
o In a typical night, a person cycles through five episodes of non-REM/REM activity
o Features of REM sleep:
1. Increased recall of dream if awoken
2. Increased sympathetic activity
3. Increased heart rate, systolic blood pressure, respiratory rate, cerebral blood flow
4. Penile erection or vaginal blood flow
5. Increased protein synthesis
6. Occasional myoclonic jerks
7. Maximal loss of muscle tone
8. Transient runs of conjugate ocular movements.
9. Nightmares occur in REM sleep – hence they are well recollected.
 Sleep spindles
 Spindles are groups of waves that occur during many sleep stages but especially in stage
2.
 They have frequencies in the upper levels of alpha or lower levels of beta.
 Duration usually <1second.
 They increase in amplitude initially and then decrease slowly. The waveform resembles a
spindle.
 They usually are symmetric and are most obvious in the parasagittal regions.
 K complex:
 K complex waves are large-amplitude delta frequency waves, sometimes with a sharp
apex.
 They can occur throughout the brain but more prominent in the bifrontal regions.
 These may be mediated by thalamo-cortical circuitry.
 Usually symmetric, they occur each time the patient is aroused partially from sleep.
 Semiarousal often follows brief noises; with longer sounds, repeated K complexes
can occur.
 K complexes sometimes are followed by runs of generalized rhythmic theta waves;
the whole complex is termed an arousal burst.
 V waves:
 V waves are sharp waves that occur during sleep. They are largest and most evident
at the vertex bilaterally and usually symmetrically.
 V waves tend to occur especially during stage 2 sleep and may be multiple.
 Often, they occur after sleep disturbances (eg, brief sounds) and, like K complexes,
may occur during brief semiarousals.
http://www.emedicine.com/neuro/TOPIC275.HTM
Sleep and age variations:
► Newborns sleep about 16 hours a day. They spend >50% of sleep time in REM sleep.
Sleep onset REM is also seen in neonates.
► By 4 months of age, the pattern shifts so that the total percentage of REM sleep drops to
less than 40 percent, and entry into sleep occurs with an initial period of NREM sleep. By
late teens adult pattern is established.

Neurosciences Answers 113


© 2010 SPMM Course www.spmmpsychiatrycourse.com

► This distribution remains relatively constant into old age with absolute reduction occurs
in both slow-wave sleep and REM sleep in older persons. An increase in awakening after
sleep onset also occurs with age.

Dreaming occurs at all stages of sleep, but the content varies. In non-REM sleep the dreams are
thought-like as though the person is solving a problem. In REM sleep the dreams are illogical and
bizarre.

Sleep regulation:
 The master clock of the brain is the suprachiasmatic nucleus (SCN) located in the anterior
hypothalamus - this orchestrates circadian rhythms and is synchronized by signals from the
retina.
 Specialized melanopsin-containing retinal ganglion cells project via retino-hypothalamic
tract to the SCN. This provides light input independent of vision.
 In the absence of solar guidance, the 24-hour sleep-wake cycle will gradually increase to
approximately 26 hours - called free-running.
 SCN is reset each day by signals of light from the retina. However melatonin secreted during
the dark cycle from the pineal gland can also reset the SCN. Hence the use of melatonin to
promote sleep in those with delayed sleep onset or jet lag.
 Sleep switch nucleus:
o The ventrolateral preoptic nuclei (VLPO) has projections to the main components of
the ascending arousal system.
o The VLPO induces sleep by putting the brakes on the arousal nuclei.
o People with damage to their VLPO have chronic insomnia.
o The VLPO must be inhibited so that people can wake up. This is possible by a
negative feedback from monoaminergic system.
o The switching is stabilised by orexin neurons (also called hypocretin) from
hypothalamus. Patients with narcolepsy have few orexin neurons in the
hypothalamus. Orexin neurons are mainly active during wakefulness and reinforce
the arousal system.
Ohayon MM, Carskadon MA, Guilleminault C, et al. Meta-analysis of quantitative sleep parameters
from childhood to old age in healthy individuals: Developing normative sleep values across the
human lifespan. Sleep. 2004; 27[7]:1255-1273.

Sleep control systems – ARAS (ascending reticular activating system)


Neurotransmitter Cell Bodies function
Cholinergic Midbrain-pons nuclei REM on neurons;

Noradrenergic Locus coeruleus REM off neurons; activation


reduces REM sleep.

Dopaminergic Periaqueductal gray matter ?D2 enhances REM

Serotoninergic Raphe nuclei ?5HT2 stimulation maintains


arousal
Histaminergic Tuberomammillary nucleus ?H1 stimulation maintains
arousal

Neurosciences Answers 114


© 2010 SPMM Course www.spmmpsychiatrycourse.com

Sleep and psychiatric disorders / drugs:


Disorder / drugs Changes
Alcohol Increase SWS (chronic use – loss of SWS)
Reduce initial REM but increase second half REM
Alcohol withdrawal Loss of SWS
Increased REM
Intense REM rebound
Anxiety disorders Increased stage 1 sleep (light sleep)
Reduced REM, normal REM latency
Reduced slow wave sleep
Benzodiazepines Decrease sleep latency
Increase sleep time
Reduce stage 1 sleep
Increase stage 2 sleep
Reduce REM
Reduce SWS
REM rebound on cessation
Cannabis Increase SWS
Suppress REM
Carbamazepine Suppresses REM
Increases REM latency
Increases SWS
Dementia Increased sleep latency
Reduced sleep time
Increased fragmentation of sleep
Depression Loss of SWS slow wave sleep (first half)
Increased REM (leading on to Early awakening)
Reduced REM latency
Lithium Suppresses REM
Increases REM latency
Increases SWS
Opiates Decrease SWS & REM
Withdrawal REM rebound
Schizophrenia Inconsistent reduction in REM latency and slow wave sleep.
(antipsychotics have variable effects)
SSRIs Alerting due to 5HT2 stimulation
May reduce REM latency
Variable effects on REM suppression
Stimulants Reduce sleep time
Decrease REM sleep and SWS
REM rebound on cessation (except modafinil)
Tricyclics REM suppression (especially Clomipramine)
Increased SWS
Increased stage 1 sleep
Z hypnotics Less effect on sleep architecture; Zopiclone may increase SWS

Electroencephalography (EEG):
 It is the recording of the electrical activity of the brain used in psychiatry primarily to
rule out seizures, monitor ECT and in polysomnogram for sleep disorders.

Neurosciences Answers 115


© 2010 SPMM Course www.spmmpsychiatrycourse.com

 Quantitative electroencephalography (QEEG) and cerebral evoked potentials (EP) are


newer EEG-based methods used in research of brain functioning.
 Standard EEG uses 21 electrodes placed on scalp. Placement of the electrodes is based
on the 10/20 International System of Electrode Placement.
 This system measures the distance between readily identifiable landmarks on the head
and then locates electrode positions at 10 percent or 20 percent of that distance in an
anterior-posterior or transverse direction.
 Activation procedures are used to bring up abnormal discharges.
 Strenuous hyperventilation (most common, safe)
 Photic stimulation using an intense strobe light
 24 hours of sleep deprivation can lead to the activation of paroxysmal EEG
discharges in some cases

 EEG recording during sleep, natural or sedated can also be used when the wake tracing
is normal.
.
Wave forms
Beta >13Hz Some seen at frontal central position in the
normal waking EEG;
Alpha 8 to 13 Hz Dominant brain wave frequency when eyes
closed and relaxing; occipito parietal
predilection. Disappears with anxiety, arousal,
eye opening or focused attention. Dominance
reduces with age.
Theta 4 to 8 Hz Small amount of sporadic theta seen in wake
EEG at fronto-temporal area; prominent in
drowsy or sleep EEG. Excessive theta in awake
EEG is a sign of pathology.
Delta <4 Hz Not seen in waking EEG. Common in deeper
stages of sleep; presence of focal/generalized
delta in awake EEG is a sign of pathology.
Mu 7-11 Hz Occurs over the motor cortex. It is related to
motor activity, characterized by arch like
waves; gets attenuated by movement of the
contralateral limb
Lambda Single a single occipital triangular, symmetrical sharp
waves wave produced by visual scanning when awake
(e.g reading) or in light sleep

 Newborns have dominant delta and theta waves. Infants have irregular medium- to
high-voltage delta activity in the awake tracing of the infant; alpha range develops in
posterior areas by early childhood; by mid-adolescence EEG essentially has the
appearance of an adult tracing. The normal dominant alpha rhythm is usually achieved
by 12-14 years old
 Diffuse slowing of background is the most common EEG abnormality; it is nonspecific
and signifies presence of encephalopathy
 Focal slowing suggest local mass lesions; e.g. edema, haematoma or focal seizure.
 Nearly 50% of those with triphasic waves have hepatic encephalopathy; the rest have
other toxic-metabolic encephalopathies.
 Beta and alpha are called fast waves; theta and delta are slow waves.

Neurosciences Answers 116


© 2010 SPMM Course www.spmmpsychiatrycourse.com

 Epileptiform discharges, when seen interictally, can be considered as hallmark of


seizure disorder. But this is not a common finding. If this is lateralized and periodic, it
may suggest an acute destructive brain lesion.
 It is also seen in generalized cerebral anoxia.

Psychotropics
Antipsychotics slowing of beta activity with increase in alpha, theta and delta activity
Antidepressants slowing of beta activity with increase in alpha, theta and delta activity
Lithium Slowing of alpha or paroxysmal activity
Anticonvulsants No effect on awake EEG

Primarily sedating drugs – decrease alpha


Barbiturates Opposite to alcohol; Increased beta activity; in withdrawal states, generalized
paroxysmal activity and spike discharges (even without overt fits)
Benzodiazepines Increased beta; decreased alpha. Overdose leads to diffuse slowing
Opioids Decreased alpha activity; increased voltage of theta and delta waves; in
overdose, slow waves

Primarily recreational drugs – increase alpha


Alcohol Increased alpha activity; increased theta activity. Withdrawal increases beta.
Delirium tremens has beta (fast) wave activity – other deliria have increased
slow waves.
Marijuana Increased alpha activity in frontal area of brain; overall slow alpha activity

Cocaine Same as marijuana; longer lasting.


Nicotine Increased alpha activity; in withdrawal, marked decrease in alpha activity

Caffeine In withdrawal, increase in amplitude or voltage of theta activity

Neurosciences Answers 117


© 2010 SPMM Course www.spmmpsychiatrycourse.com

EEG in various disorders


Absence seizures (petit-mal) Regular 3 Hz Complexes
Alzheimer’s dementia Rarely normal in advanced dementia; may be helpful in
differentiating pseudodementia from dementia
Angelman’s syndrome 1. EEG changes are notable by the age of 2.
2. Prolonged runs of high amplitude 2–3 Hz frontal
activity with superimposed interictal epileptiform
discharges – all ages
3. Occipital high amplitude rhythmic 4–6 Hz
activity facilitated by eye closure, is seen under
the age of 12 years.
4. There is no difference in EEG findings in AS
patients with or without seizures
Antisocial personality disorder Increased incidence of EEG abnormalities in those with
aggressive behaviour
Attention-deficit/hyperactivity up to 60% have EEG abnormalities (spike/spike-waves)
disorder (ADHD)
Borderline personality Positive spikes: 14- and 6 per second seen in 25% of
disorder patients
CJD Generalised periodic 1-2 Hz sharp waves are seen in
nearly 90% patients with sporadic CJD. Less often in
familial / hormonal transplant related forms. NOT seen in
variant form.
Closed head injuries Focal slowing (sharply focal head trauma)
Focal delta slowing (subdural hematomas)
Diffuse atherosclerosis Slowed alpha frequency and increased generalized theta
slowing
Herpes simplex encephalitis Episodic discharges recurring every 1-3 seconds with
variable focal waves over the temporal areas.
Huntington’s dementia Initially loss of alpha; later flattened trace
Infantile spasms (seen in Hypsarrhythmia [diffuse giant waves (high voltage, >400
tuberous sclerosis) microvolts) with a chaotic background of irregular,
asynchronous multifocal spikes and sharp waves].
Clinical seizures are associated with a marked
suppression of the background - called the
electrodecremental response
Infectious disorders Diffuse, often synchronous, high voltage slowing (acute
phase of encephalitis)
Metabolic and endocrine Diffuse generalized slowing. Triphasic waves: 1.5 to 3.0
disorders per second high-voltage slow-waves especially in hepatic
encephalopathy.
Neurosyphilis Non-specific increase in slow waves occurring diffusely
over the scalp.
Panic disorder Paroxysmal EEG changes consistent with partial seizure
activity in one-third; focal slowing in about 25% of
patients
Seizures Generalized, hemispheric, or focal spike/ spike-wave
discharge.
Stroke Focal or regional delta activity

Neurosciences Answers 118


© 2010 SPMM Course www.spmmpsychiatrycourse.com

Structural lesions Focal slowing / focal spike activity

S.G. Boyd, A. Harden and M.A. Patton, The EEG in early diagnosis of the Angelman (happy puppet)
syndrome. Eur J Pediatr 147 (1988), pp. 508–513

Event Related Potential (ERP):


► An ERP is a change in electrical brain activity stereotyped and time-locked to an event
(e.g., stimulus), although it can also occur for omission of an expected stimulus. This
allows the investigation of specific types of information processing by the brain.
► ERPs are small relative to the spontaneous brain activity (background EEG) that is they
have a low signal-to-noise ratio.
► To increase the signal-to-noise ratio an often-used method is ERP averaging
► ERPs are characterized by
o Polarity (positive or negative - indicated by the letter in the ERP name)
o Latency (the moment of peak occurrence after stimulus presentation, which is
often indicated with the number in the ERP name) and
o Scalp distribution.
► The temporal resolution of ERP analysis is much higher than that of other neuro-imaging
methods like functional MRI, SPECT and PET (i.e., it is in the order of milliseconds
► ERP analysis lacks spatial resolution compared to the MRI techniques.
► According to time of occurrence ERPs can be
o Early: Basic sensory pathways can be studied by recording of early ERPs or
‗evoked potentials‘ (EPs, brain stem evoked responses- BAER) in response to
sounds (Auditory EP, AEP), flashes (Visual EP, VEP) or electrical stimulation
(Somatosensory EP, SEP).
o Midlatency evoked potentials occur after BAER. The three well known are
N100, P50 and P200. Their amplitudes reduce with repetition – habituation
response / sensory gating.
o Late: Cognitive pathways can be studied by recording of ERPs related to the
execution of psychological events such as attention, emotion or memory tasks.
These are the most studied in psychiatry as discussed below.
► The P300, a positive ERP component around 300 ms after stimulus presentation, is
typically generated when a rare target stimulus is imbedded with more frequent stimuli
e.g. (auditory ‘oddball’ protocol). The P300 is considered to be related to the
maintenance of working memory. Decrease in P300 amplitude is well established as a
biological trait marker in schizophrenia.
► The Mismatch Negativity or MMN is a negative ERP component that is recorded
between 100-200 ms in response to low-probability deviant sounds in a sequence of
standard sound stimuli, when the participant is not actively attending to the deviants. The
MMN is best seen in the difference wave between the ERP in response to the standard
and deviant sounds. The MMN reflects involuntary information processing in auditory
context, i.e. the mnemonic comparison of a given stimulus with a previous one which has
already built up a trace in memory. The violation of the previously formed memory trace
produces the MMN. Decreased MMN amplitude is noted in schizophrenia.
► The Contingent Negative Variation (CNV) is noted in the recording of the brain
response to a warned reaction time task. CNV reduction is noted in schizophrenia patients
especially with long duration of illness with positive symptoms.

Kaplan & Sadock comprehensive textbook – Page 199-200.

Neurosciences Answers 119


© 2010 SPMM Course www.spmmpsychiatrycourse.com

Magnetic electroencephalography:
o Magnetoencephalography (MEG) is used to measure the magnetic fields produced by
electrical activity in the brain via extremely sensitive devices such as superconducting
quantum interference devices.
o In contrast to electric fields, magnetic fields are less distorted by the skull and scalp. The
scalp EEG is sensitive to both tangential and radial components of a current source in a
spherical volume conductor, MEG detects only its tangential components.
o This shows MEG selectively measures the activity in the sulci, whereas scalp EEG
measures activity both in the sulci and at the top of the cortical gyri.

Brain stimulation therapies:


Transcranial magnetic stimulation:
Repetitive transcranial magnetic stimulation (rTMS) is a non-invasive technique used to
stimulate the human brain in vivo using very strong, pulsed magnetic fields.
The technique involves the delivery of a magnetic pulse to the cortex of a subject through
a hand-held stimulating coil applied directly to the head. The magnetic pulses pass
unimpeded through the skull and induce an electrical current in the underlying tissue,
which in turn is able to depolarise neurons.
It may have therapeutic effects in affective disorders similar to ECT. Few studies have
reported a beneficial effect of left prefrontal rTMS in severe depression.
Martin, J. L. R., Barbanoj, M. J., Schlaepfer, T. E., et al (2003) Repetitive transcranial
magnetic stimulation for the treatment of depression: systematic review and meta-analysis.
British Journal of Psychiatry, 182, 480 -491.

Vagus Nerve Stimulation - VNS:


Vagus Nerve Stimulation refers to stimulation of the left cervical vagus nerve
It is available for the treatment of resistant partial-onset seizures in epilepsy
The basic mechanisms of action of VNS are unknown. It may result in changes in
serotonin, norepinephrine, GABA, and glutamate neurotransmitters implicated in the
pathogenesis of major depression.
George, M. S., Sackeim, H. A., Rush, A. J., et al (2000) Vagus nerve stimulation: a new tool
for brain research and therapy. Biological Psychiatry, 47, 287 -295

Physiological changes after ECT:


 EEG shows delta and theta – returns normal after 3 months of the end of
treatment
 May increase BBB permeability.
 Increased secretion of ADH, ACTH, and endorphins noted.
 Increase in 5HT2 receptors – different from antidepressant effect.
 Reduced β noradrenergic receptors.
 Increased noradrenaline turnover noted.
 Alpha 2 receptors reduced similar to antidepressants.

Neurosciences Answers 120


© 2010 SPMM Course www.spmmpsychiatrycourse.com

Neuroimaging:
Computed Tomography – CT:
 The most widely available scan in clinical practice
 CT scanners effectively take a series of head X-ray pictures from 360 degrees around a
patient's head.
 The CT image is determined by the degree to which tissues absorb X-rays.
 The structures close to bone may be obscured; e.g. brainstem,
 The difference in the attenuation between gray matter and white matter is not very high.
 CT is limited to one plane of rotation – often axial.
 Appreciation of tumours and areas of inflammation is possible by intravenous infusion of
iodine-containing contrast agents. Iodinated compounds absorb much more irradiation
than the brain and so appear white.
 The only component of the brain better seen on CT scanning is calcification, which may
be invisible on MRI.

Magnetic Resonance Imaging – MRI:


 MRI does not rely on the absorption of X-rays but is based on nuclear magnetic
resonance (NMR).
 The nuclei of all atoms spin about an axis that is randomly oriented in space. When atoms
are placed in a magnetic field, the axes of all odd-numbered nuclei (H1 commonly) align
with the magnetic field. This axis deviates away from the magnetic field when exposed to
a pulse of radiofrequency electromagnetic radiation oriented at 90 or 180 degrees to the
magnetic field. When the pulse terminates, the axis of the spinning nucleus realigns itself
with the magnetic field, and during this realignment, it emits its own radiofrequency
signal. MRI scanners collect the emissions of these signals.
 The images can be in the axial, coronal, or sagittal planes.
 The rate of realignment of the H1 axis is determined by its immediate environment and
the degree of water content.
 Hydrogen nuclei within fat realign rapidly, and hydrogen nuclei within water realign
slowly.
 Routine MRI studies use three different radiofrequency pulse sequences.
o T1 images:
 The RF pulses are brief and data collection is brief;
 Hydrophobic environments are emphasized i.e fat is bright on T1, and
CSF is dark.
 The T1 image most closely resembles that of CT scans and is most useful
for assessing overall brain structure.
 T1 is also the only sequence that allows contrast enhancement with the
contrast agent gadolinium-diethylenetriamine pentaacetic acid
(gadolinium-DTPA).
 On T1 images, gadolinium-enhanced structures appear white.
o T2 images:
 The RF pulse last four times as long as T1 pulses, and the collection
times are also extended.
 Empasizes signal from hydrophilic areas i.e. brain tissue is dark, and
CSF is white on T2 images.
 Areas within the brain tissue that have abnormally high water content,
such as tumors, inflammation, or strokes, appear brighter on T2 images.
T2 images reveal brain pathology most clearly.

Neurosciences Answers 121


© 2010 SPMM Course www.spmmpsychiatrycourse.com

o The proton density sequence:


 A short radio pulse is followed by a prolonged period of data collection,
 Useful to see periventricular structures
o Fluid-attenuated inversion recovery (FLAIR):
 T1 image is inverted and added to the T2 image to double the contrast
between gray matter and white matter.
 Very useful for detecting sclerosis of the hippocampus caused by
temporal lobe epilepsy and for localizing areas of abnormal metabolism
in degenerative neurological disorders.
 MRI magnets are rated in teslas (T), units of magnetic field strength.
 MRI scans cannot be used for patients with pacemakers or implants of ferromagnetic
metals.
 Claustrophobia is a contraindication.
Structures / pathology CT scan T1 image T2 image
infarct Dark Dark Bright
Bleed (haemorrhage) Bright Bright (unless too Bright (unless
old / too fresh) too old / too
fresh)
Tumour Dark Dark Bright
MS plaque Dark Dark Bright
(demyelinated)
CSF Dark Dark Bright
Bone Bright Bright Dark
Air Dark Dark Dark
Fat Dark Bright Bright
Tissue Shades of grey Grey matter – grey Shades of grey
White matter -
white

Magnetic Resonance Spectroscopy –MRS:


 MRI detects hydrogen nuclei - MRS can detect several biologically important nuclei MRS
can image nuclei with an odd number of protons and neutrons.
 The MRS of the hydrogen-1 nuclei is best at measuring N-acetylaspartate (NAA), creatine,
and choline-containing molecules.
 GABA and glutamate can be detected but not dopamine as it is available ina very low
amount
 MRS of phosphorus-31 can be used to determine the pH of brain regions and the
concentrations of phosphorus-containing compounds (e.g., adenosine triphosphate [ATP]
and guanosine triphosphate [GTP]), which are important in the energy metabolism of the
brain.
 Additional indications include the use of MRS to measure concentrations of
psychotherapeutic drugs in the brain. Some compounds, such as fluoxetine and
trifluoperazine (Stelazine), contain fluorine-19, which can also be detected in the brain and
measured by MRS.
.
Functional Magnetic Resonance Imaging (fMRI)
 Neuronal activity within the brain causes a local increase in blood flow, which in turn
increases the local hemoglobin concentration. Although neuronal metabolism extracts
more oxygen in active areas of the brain, the net effect of neuronal activity is to increase
the local amount of oxygenated hemoglobin. This change can be detected with the T2

Neurosciences Answers 122


© 2010 SPMM Course www.spmmpsychiatrycourse.com

sequence, called Blood Oxygen Level Dependent (BOLD) technique. This process is the
basis for the technique of fMRI.
 The method detects tissue perfusion, not neuronal metabolism.
 No radioactive isotopes are administered in fMRI, a great advantage over PET and
SPECT.
 A subject can perform a variety of tasks, both experimental and control, in the same
imaging session.

Nucleus Potential Clinical Uses


1
H Magnetic resonance imaging (MRI)
Analysis of metabolism – NAA, creatine and choline.

19
F Measurement of pO2
Analysis of glucose metabolism
Measurement of pH
Pharmacokinetics
7
Li Pharmacokinetics
31
P Analysis of bioenergetics
Measurement of pH
14
N Measurement of glutamate, urea, ammonia
13
C Analysis of metabolite turnover rate
Pharmacokinetics of labeled drugs
17
O Measurement of metabolic rate

Single Photon Emission Computed Tomography - SPECT:


o SPECT uses radioactive compounds to study regional differences in cerebral blood flow
within the brain. This records the pattern of photon emission from the bloodstream
according to the level of perfusion in different regions of the brain.
o As with fMRI it does not measure neuronal metabolism directly.
o SPECT uses compounds labeled with single photon-emitting isotopes: iodine-123,
technetium-99m, and xenon-133.
o Xenon-133 quickly enters the blood and is distributed to areas of the brain as a function
of regional blood flow. Xenon-SPECT is thus referred to as the regional cerebral blood
flow (rCBF) technique. Xenon-SPECT can measure blood flow only on the surface of the
brain, which is an important limitation.
o Assessment of blood flow over the whole brain with SPECT requires the injectable
tracers, technetium-99m-d,l-hexamethylpropyleneamine oxime (HMPAO).
o This is attached to highly lipophilic molecules that rapidly cross the blood-brain barrier
and enter cells. Once inside the cell, the ligands are enzymatically converted to charged
ions, which remain trapped in the cell. Thus, over time, the tracers are concentrated in
areas of relatively higher blood flow. This is the ligand mostly used in dementia scans for
perfusion changes.
o Iodine-123 (123I)-labeled ligands for the muscarinic, dopaminergic, and serotonergic
receptors can be used to study these receptors by SPECT technology. Iodobenzamide is
used for D1/D2 receptors; iomazenil is used for GABA-A receptors; nor-β- CIT for
dopamine and serotonin transporters; epidepride for D2/D3 receptors.

Neurosciences Answers 123


© 2010 SPMM Course www.spmmpsychiatrycourse.com

Positron Emission Tomography – PET:


o A key difference between SPECT and PET is that in SPECT a single particle is emitted,
whereas in PET two particles are emitted; the latter reaction gives a more precise location
for the event and better resolution of the image.
o The isotopes used in PET decay by emitting positrons, with the resolution closer to its
theoretical minimum of 3 mm.
o Relatively few PET scanners are available because they require an on-site cyclotron to
make the isotopes.
o The most commonly used isotopes in PET are fluorine-18, nitrogen-13, and oxygen-15.
These isotopes are usually linked to another molecule, except in the case of oxygen-15
(15O).
o The most commonly reported ligand has been [18F]fluorodeoxyglucose (FDG). This
gives information directly about neuronal metabolism.

Receptors PET ligand


Blood flow C15/H215O
Glucose metabolism F18 deoxyglucose
11
Dopamine D2 C raclopride
18
Dopamine neurn density F dopa; 18F metatyrosine
11
Benzodiazepines C flumazenil
18
5HT2 F altanserin; setoperone
11
Striatal D2, cortical 5HT2 C methylspiperone
11
Serotonin synthesis rate measurement C methyltryptophan
11
Muscarinic C scopolamine

Diffusion tensor imaging – DTI:


► DTI combines the principles of nuclear magnetic resonance and molecular diffusion.
► Diffusion refers to the random translational motion of molecules, also called Brownian
motion, that result from the energy carried by these molecules.
► During their random, diffusion-driven displacements, molecules probe tissue structure at
a microscopic scale well beyond the usual image resolution: during typical diffusion
times of about 50 msec, water molecules move in the brain on average over distances
around 10 m, bouncing, crossing, or interacting with many tissue components.
► In traditional diffusion weighted images only 3 gradient directions are applied; DTI –
diffusion tensor allows multiple (say 16) gradients .
► From DTI, diffusion anisotropy measures such as the Fractional Anisotropy (FA), can
be calculated.
► The principal direction of the diffusion tensor can be used in tractography to infer the
white-matter connectivity of the brain.

Koyama T, Tamai K, Togashi K (2006) Current status of body MR imaging : fast MR imaging and
diffusion-weighted imaging. Int J Clin Oncol 11:278-285.

Neurosciences Answers 124


© 2010 SPMM Course www.spmmpsychiatrycourse.com

Neuropsychology:
Parietal lobe functioning
Subtest Comments
Alexia & Agraphia Dominant, part of Gerstmann syndrome. Intact speech but cannot read or
write meaningful language
Anosognosia Patient does not recognise functional deficits of their body. Non dominant
lobe.
Calculation ability Dominant, part of Gerstmann syndrome. Involves mathematical functions.
Mere recognition and use of numbers is called arithmetic ability.
Constructional Inability to draw shapes or construct geometrical patterns. Mostly
apraxia nondominant.
Finger agnosia Dominant, part of Gerstmann syndrome.
Graphesthesia Ability to recognise what number or alphabet is scratched on one‘s skin
without seeing.
R-L orientation Dominant, part of Gerstmann syndrome.
Stereognosis Ability to recognise objects by palpation, and without visual inspection.
Two point Cortical sensation; Somatosensory cortical function
discrimination
Visual field defects Inferior quadrantanopia is seen on opposite side of lesion. (note: both
inferior and superior quadrantanopia are more common in occipital optic
radiation lesions)

Frontal lobe tests:


Subtest Comments
Similarities Compare two objects – look for ‗categorisation‘ and not description of
common ‗parts‘. Conceptualisation of the whole objects is more important
for abstract ability.
Lexical Can use supermarket list generation or FAS test. Tests not only the speed and
fluency accuracy but also the ability to shift from a narrow set to the next. E.g.
supermarket list must include not only fruits, but also bakery items,
households etc.
Luria motor Fist palm edge – must not be verbally facilitated. Test for motor planning,
test execution and error correction.
Go/on go test Tests response inhibition, absence of perseveration and resistance to
interference.
Cognitive e.g. ‗How tall is an average English woman?‘ Use questions that need
estimate abstract not mere factual thinking.
testing
Trail making Consists of two parts. In part A simple number sequence is used to join the
test: dots. Test B uses alternating numbers and letters and is thought to be more
sensitive for frontal lobe dysfunction. Not specific for frontal lobe; tests
visuomotor tracing, attention, conceptualisation and set shifting.
Other tests Include alternate pyramids and squares drawing, proverb interpretation, and
to some extent frontal release signs and digit span (normal: 7±2 forwards,
5±1 backwards) reflect frontal functions.

Neurosciences Answers 125


© 2010 SPMM Course www.spmmpsychiatrycourse.com

Semple et al (Ed). The Oxford Handbook of Psychiatry 1 st edition. Oxford University Press 2005.

Psychometry:
It is important to be familiar with certain terms used in psychometry:

Reliability: extent to which a test can produce results that are consistent across various raters
(inter-rater reliability) or various instances when used by the same rater (test-retest reliability)

Validity: extent to which a test measures what it is supposed to measure


Construct validity: extent to which performance on a test fits with theoretical schema
about the test attributes
Predictive validity: ability of test result to predict external criterion that will take place in
future
Incremental validity: what the test adds to the predictive validity of already existing
standard tests
Criterion validity: a test‘s correlation with similar measures – called concurrent validity if
this is applied at the same time

The Wechsler Adult Intelligence Scale (WAIS):


o Most widely used intelligence test in clinical practice.
o The latest revision, the WAIS-III, is designed for persons 16 to 89 years of age. Wechsler
Intelligence Scale for Children-III [WISC-III] is used for <16.
o For ages 4 to 61/2 years Wechsler Preschool and Primary Scale of Intelligence-Revised
[WPPSI-R] is used.
o The WAIS is composed of 11 subtests made up of six verbal subtests and five performance
subtests, which yield a verbal IQ, a performance IQ, and a combined or full-scale IQ.
o Verbal tests = similarities, arithmetic, digit span, vocabulary, information and
comprehension
o Performance tests = picture arrangement, block design, picture completion, digit symbol,
matrix reasoning (replaces object assembly)
o In WAIS hold tests are vocabulary, information, object assembly and picture completion.
Non hold tests are block design, digit span, similiarities and digit symbol. A deterioration
quotient is derived from the difference between ‗don‘t hold‘ and ‗hold‘ test scores.
o Hold tests are supposed to be resistant to age related decline; so they may be sensitive for
organic brain damage such as dementia. But this is refuted.
Raven’s progressive matrices:
IQ test
Independent of education and cultural influence
Taps on general intelligence with visuospatial problem solving.
NART – National adult reading test:
Taps on previous word knowledge before becoming ill – hence used to estimate premorbid IQ.
Stroop test:
Measures set shifting abilities and response inhibition
Test of frontal function and slective attention ability.
The Wisconsin Card Sorting Test (WCST) contains stimulus cards of different colour,
form, and number. These are presented to patients to sort into groups according to a single
principle (e.g., to sort by colour, ignoring form and number). Persons with damage to the frontal
lobes or to the caudate and some persons with schizophrenia give abnormal responses.

Neurosciences Answers 126


© 2010 SPMM Course www.spmmpsychiatrycourse.com

Trail Making Test (TMT - B), Wisconsin Card Sort Test (WCST), Hayling test
(Sentence completion), Brixton task, all test set-shifting ability which is a part of
executive functioning.

The Wechsler Memory Scale-Revised (WMS-R):


 It is the most widely used memory test battery for adults.
 It contains
o verbal paired associate
o paragraph retention,
o visual memory for designs,
o orientation,
o digit span,
o rote recall of the alphabet, and
o counting backward.
 The scale yields a memory quotient (MQ), which is corrected for age and generally
approximates the WAIS IQ.
 In amnesic conditions a disproportionately low MQ but a relatively preserved IQ is seen.
Benton Visual Retention Test: This involves the presentation of each geometric figure for 10
seconds, after which the patient attempts to draw the figure from memory. (Short term memory
test)
The Bender Visual Motor Gestalt Test is a test of visuomotor coordination that is useful for
both children and adults.
Halstead & Reitan developed a battery of tests that was used to determine the location and the
effects of specific brain lesions.
It includes
 Category test: Patients must discover the common element in a set of pictures
 Tactual performance test: spatial memory, and tactual discrimination.
 Rhythm test: auditory perception, attention, and concentration.
 Finger-oscillation test: dexterity and motor speed.
 Speech-sounds perception test: auditory discrimination and phonetic skills.
 Trail-making test A and B: visuomotor perception and motor speed. B requires set
shifting.
 Critical flicker frequency: visual perception.
 Time sense test: memory and spatial perception.
 Aphasia screening test: wide range of verbal and nonverbal brain functions.
 Sensory-perceptual tests: simultaneous sensory stimulation test for finger localization,
stereognosis and graphesthesia.

Schizophrenia:
Studies on populations with schizophrenia presenting with predominantly negative symptoms
indicate an overall deterioration in IQ, short-term memory disturbances, deficits in higher order
reasoning and perceptual difficulties.
There is a tendency for people with schizophrenia to perform poorly on tests that assess frontal
and temporal function. In addition there is evidence to indicate specific disturbances of short-term
memory, especially verbal short-term memory.
They perform far more poorly than controls on category test (assess higher-order reasoning),
Wisconsin card sort test (assess higher-order reasoning), Speech sounds perception test, Trail B
of the Halstead-Reitan battery (assess ability to shift flexibility between cognitive sets)

Neurosciences Answers 127


© 2010 SPMM Course www.spmmpsychiatrycourse.com

Behavioural & clinical Neurology:


CONSCIOUSNESS:
 Consciousness is a state characterised by awareness of self and environment and an ability to
respond to environmental factors; it is made up of two components – arousal (wakefulness)
and awareness (information processing).
 Arousal depends on intact functioning of ARAS – Ascending Reticular activating System.
 Thalamo-cortical connections generate rhythmical bursts of neuronal activity (20 – 40 Hz)
which are in desynchrony by default. ARAS acting via the thalamic intralaminar nuclei
synchronises these oscillations. Wakefulness is directly proportional to the degree of such
synchrony achieved.
 The maintenance of attention appears to require an intact right frontal lobe
 The absence of arousal produces stupor and coma.
 Small lesions of ARAS are enough to produce a stuporous state, but large bilateral lesions are
required in the cortical level to cause the same depression in alertness.

Stupor: In this state the individual appears to be asleep and yet, when vigorously stimulated, may
become alert as manifest by eye opening and ocular movement.
Most patients in stupor have diffuse organic cerebral dysfunction. Caloric testing in organic
stupor will usually reveal tonic deviation, whereas in psychiatric stupor (catatonia/depression)
ocular nystagmus will be present. This is because the fast phase reflects the correction following
tonic deviation and this requires the patient to be conscious.

Akinetic mutism: It was first described in patients thought to have suffered diencephalic damage.
The syndrome is characterised by immobility and eye closure with little or no vocalisation.
Sleep/wake cycles can be seen, as indicated by eye opening. There is little in the way of
movement to painful stimuli and the hallmark is the relative paucity of signs indicating damage to
descending motor pathways, despite the immobile state. In other words spasticity and rigidity are
not usually evident. Akinetic mutism can arise as a result of lesions that interfere with reticular
cortical/integration but spare the corticospinal pathways. There is some debate about whether or

Neurosciences Answers 128


© 2010 SPMM Course www.spmmpsychiatrycourse.com

not the syndrome should be clearly differentiated from the vegetative state. CJD is well known to
result in Akinetic mutism before death.

Vegetative state:
This condition represents the expression of the isolated actions of the ARAS and the thalamus,
free of cortical influence due to extensive cortical damage.
A patient in the fully established vegetative state will almost invariably show spasticity and
rigidity of the limbs which are absent from patients with the syndrome of akinetic mutism. In the
early stages of the vegetative state, the two clinical syndromes are indistinguishable.
The Royal College of Physicians produced a series of clinical criteria.
1. There should be no evidence of awareness of self or environment at any time.
2. There should be no volitional response to visual auditory, tactile or noxious
stimuli.
3. There should be no evidence of language comprehension or expression.
4. There should be presence of cycles of eye closure and eye opening which may
simulate sleep and wakening.
5. There should be sufficiently preserved hypothalamic and brain stem function to
ensure the maintenance of respiration and circulation.
The clinical picture may be seen as a transient phase during the recovery of acute brain damage or
may be permanent as a consequence of the failure to recover from brain damage.
Locked in syndrome: The ventral pontine or locked in syndrome describes a condition of total
paralysis below the level of the third nerve nuclei. Such patients can open their eyes and elevate
and depress their eyes to command. Horizontal eye movements are usually lost and no other
voluntary movement is possible. The diagnosis of this state depends on the recognition that the
patient can open his eyes voluntarily rather than spontaneously in the vegetative state. The
neuropathological basis for this condition is usually infarction of the ventral pons and efferent
motor tracks. A similar clinical picture may sometimes be seen in patients with pontine tumours,
pontine haemorrhage, central pontine myelinolysis, head injury or brain stem encephalitis.

Cartlidge, N. States related to or confused with coma. Neurol Neurosurg Psychiatry 2001; 71(Suppl
1):i18-i19

MEMORY:

► Classification of memory:
o According to duration:
 Immediate memory functions over a period of seconds; closely related to
concept of working memory
 Recent memory applies on the scale of minutes to days; and
 Remote memory encompasses months to years.

o According to encoding memory can be classified into explicit or declarative memory


and implicit or procedural memory.
 Explicit memory can be either semantic (meanings) or episodic (events).
 Implicit memory consists of skills and procedures e.g. car driving.

► Brain structures involved in memory:


o Hippocampus:
 Left for declarative verbal and right for encoding non verbal memories.
 Navigational memory and memory of object location in space are served by
hippocampus.

Neurosciences Answers 129


© 2010 SPMM Course www.spmmpsychiatrycourse.com

 Animal studies have defined a hippocampal place code, a pattern of cellular


activation in the hippocampus that corresponds to the animal's location in
space.
 Unilateral hippocampal lesions are compensated well and clinically
significant amnesia does not occur.
o Amygdala has been suggested to rate the emotional importance of an experience and
to activate the level of hippocampal activity accordingly. It is involved in emotional
memory and emotional face processing. It helps in memory consolidation, depending
on emotional input for the content of memory. Amygdalar damage leads to loss of
fear conditioning and in monkeys, loss of maternal behaviour has also been noted.
Despite amygdalar damage, learning and consolidation of memory can occur,
especially in the absence of emotional valence and arousal.
o Diencephalic structures such as the dorsal medial nucleus of the thalamus and the
mamillary bodies are associated with new learning; their damage leads to
diencephalic amnesia seen in Korsakoff syndrome.
► In most cases of memory loss procedural memory is intact. A deficit in procedural memory
with preservation of declarative memory may be seen in persons with Parkinson's disease, in
whom dopaminergic neurons of the nigrostriatal tract degenerate. Though speculative,
cerebellum, striatum, amygdala and certain parts of neocortex (including motor area) are
thought to be involved in non-declarative procedural memry storage.
► Strengthening of the connection between two neurons on repeated communication is called
long-term potentiation - LTP. This may be the neuronal basis of memory. It is mediated by
NMDA-CA2+ in glutamate neurons. Learning increases branching and synapse formation and
may also influence neurogenesis.

Disorders of memory:
 Amnesia is a term used for pure memory deficits (mostly episodic) or cognitive deficits
where memory loss is predominant and not congruent with the level of loss in other
domains.
 Episodic memory depends on the hippocampal–diencephalic system. It is the time-
locked memory for personal events (‗when and where‘ memory); it includes both
anterograde and retrograde memory.
Anterograde - newly encountered information from the time of lesion.
Anterograde memory loss is commonly elicited from the history in a dementing patient -
forgetting appointments, losing items around the home, inability to remember conversation
leading to repeated questions etc.
Retrograde - memory of past events that happened before the lesion was sustained.
Retrograde memory loss is evident from a history of loss of memory of past events such as
jobs, holidays, not able to remember the topography of a route and getting lost.
 Generally both anterograde and retrograde memory loss occur in parallel, such as
in Alzheimer‘s disease or head injury.
 Relatively pure anterograde amnesia may be seen when there is hippocampal
damage, e.g. herpes simplex encephalitis, focal temporal lobe tumours, or
infarction.
 Confabulation—for example, in Korsakoff‘s syndrome—might be grandiose or
delusional, but more often involves the mis-ordering and fusion of real memories
which end up being retrieved out of context.
 A transient amnesic syndrome with pronounced anterograde, and variable
retrograde, amnesia is seen in transient global amnesia (TGA), while ‗‗memory
lacunes‘‘, and repeated brief episodes of memory loss suggest transient epileptic
amnesia (TEA).

Neurosciences Answers 130


© 2010 SPMM Course www.spmmpsychiatrycourse.com

 Ribot's Law of retrograde amnesia: ‗The dissolution of memory is inversely


related to the recency of the event‘. Recent memories are more likely to be lost
than the more remote memories in organic amnesia (not always the case though).

 A second important system involves memory for word meaning and general knowledge
(semantic memory), the key neural substrate being the anterior temporal lobe.
 Patients with semantic breakdown typically complain of loss of words.
 Vocabulary diminishes and patients substitute words like ‗‗thing‘‘.
 There is a parallel impairment in appreciating the meaning of individual words
which first involves infrequent or unusual words.
 Word finding difficulty is common in both anxiety and aging, but variable and
not associated with impaired comprehension. This is in stark contrast to the
anomia in semantic dementia which is relentlessly progressive and associated
with atrophy of the anterior temporal lobe, usually on the left.

 Working memory refers to the very limited capacity which allows us to retain
information for a few seconds, and uses the dorsolateral prefrontal cortex. (The term
‗‗short term‘‘ memory is applied, confusingly, to a number of different memory
problems, but has no convincing anatomical or psychological correlate). It is made of a
central executive system (attentional system, prefrontal) and at least 2 important buffer
systems – the visuospatial sketch pad (right hemisphere) and phonological loop (left
hemisphere).
 Working memory deficits can present as lapses in concentration and attention
e.g. losing one‘s train of thought, inability to process a complex task as the
components is not retained long enough in memory to be processed.
 Basal ganglia and white matter diseases may present with predominantly working
memory deficits.

 Dissociative amnesia is not an organic syndrome, but centred on loss of memory of


important recent events which is partial, patchy and selective. It can occur as a part of
fugue wherein dissociative fugue is diagnosed. The characters of dissociative amnesia are
 Episodic memory loss, retrograde only – no anterograde deficits.
 Amnesia is for events that happened at a discrete period of time from minutes to
years
 The problem is not vague or inefficient retrieval but strikingly complete
unavailability of memories which were formed normally and were previously
accessible. These events are generally traumatic or stressful.

ORIENTATION:
 Orientation is usually assessed to time, place and person; it is not particularly sensitive,
and intact orientation does not exclude a significant memory disorder, particularly if there
is concern about memory from an informant.
 Time orientation is the most helpful, and should include the time of day.
 Many normal people do not know the exact date, and being out by two days or less is
considered normal when scoring this formally.
 Time intervals are often poorly monitored by patients with delirium, moderate to severe
dementia, and in the amnesic syndrome, and are easily tested by asking about the length
of time spent in hospital.
 Person orientation includes name, age, and date of birth. Disorientation to one‘s own
name is usually only seen in psychogenic amnesia.

Neurosciences Answers 131


© 2010 SPMM Course www.spmmpsychiatrycourse.com

ATTENTION:
 Attention can be tested in a number of ways including serial 7s, digit span, spelling
‗‗world‘‘ backwards, and recitation of the months of the year in reverse order.
 Although serial 7s is commonly used, it is frequently performed incorrectly by the
elderly, as well as by patients with impaired attention.
 A reverse-order month of the year is a highly over learned sequence, and is a preferred
measure of sustained attention.
 Digit span is a relatively pure test of attention, and is dependent on working memory, but
it is not specific, and can be impaired in delirium, focal left frontal damage, aphasia, and
moderate to severe dementia, but should be normal in the amnesic syndrome (for
example, Korsakoff‘s syndrome or medial temporal lobe damage).
o Start with three digits, and ensure that they are spoken individually and not
clumped together in the way that one might recite a telephone number (for
example, 3-7-2-5 and not 37-25, etc).
o Normal digit span is 7 +/- 2 depending on age and general intellectual ability. In
the elderly, or intellectually impaired, 5 can be considered normal.
o Reverse span is usually one less than forward span.

LANGUAGE:
 Aphasia refers to a higher level ‗language‘ problem – not sound production or
manipulation error but problem of language reception, production and processing.
Aphasia is almost always organic.
 To understand aphasia, consider the following facts;
 Sound received by ears is transmitted to Wernicke‘s area and auditory
association cortex which processes the language component.
 Arcuate fasciculus connects Wernicke‘s area to Broca‘s area. (NOTE:
do not confuse this with uncinate fasciculus - The uncinate
fasciculus interconnects the anterior temporal and inferior frontal
gyrus)
Broca‘s area is the higher motor area of language production. Signals
from Broca‘s area are relied onto motor area to coordinate the delivery
of language via tongue, lips and vocal cords.
 Three important components of language are
 Fluency: depends on intact Broca‘s area and its forward connections.
 Comprehension: depends on intact Wernicke‘s area and its connection
with association cortex and sensory input
 Repetition: requires no high level processing; can take place if Broca‘s ,
Wernicke‘s and arcuate fasciculus are intact. It does not need relay of
higher association area to either Broca‘s or Wernicke‘s.
Naming defects (anomia) accompanies any aphasia in various degrees.
Type of aphasia Fluency Repetition Comprehension Naming
Wernicke’s Intact Lost Lost Lost
sensory aphasia
Broca’s motor Lost Lost Intact Lost
aphasia
Conduction Intact Lost Intact Lost
aphasia
Transcortical Intact Intact Lost Lost
sensory aphasia

Neurosciences Answers 132


© 2010 SPMM Course www.spmmpsychiatrycourse.com

Transcortical Lost Intact Intact Lost


motor aphasia
Adapted from Harrison‘s Textbook of internal medicine; 15 e

In Broca's aphasia the speech is nonfluent; it often appears laboured with any interruptions and
pauses. Function words (propositions, conjunctions) are most affected though good degree of
meaning-appropriate nouns and verbs are still produced. Abnormal word order and a
characteristic agrammatism are noted. Speech is telegraphic. Harrison textbook quotes the
following example: "I see...the dotor, dotor sent me...Bosson. Go to hospital. Dotor...kept me
beside. Two, tee days, doctor send me home.
In Wernicke's aphasia the comprehension is impaired for both spoken and written language.
Language output is fluent but is highly paraphasic, sometimes with string of neologisms and
circumlocutions. Hence it is also termed as "jargon aphasia." Speech contains large numbers of
function words (e.g., prepositions, conjunctions) but few substantive nouns or verbs that refer to
specific actions. The output is therefore voluminous but uninformative.
Pure word deafness: Patient can speak read & write fluently, but comprehension is impaired
only for spoken language. Bilateral (or left sided with disrupted connections to non-dominant
circuit) damage to superior temporal pole is suspected.
Pure word blindness (alexia no agraphia): Here the patient can speak normally and
comprehend what is spoken; he can also write spontaneously and to dictation, but reading
comprehension is impaired.
Pure word dumbness: Spoken language cannot be produced clearly; but the patient can
comprehend language well, can read and write.
Pure agraphia: This is an isolated inability to write while other faculties of language are
preserved.
Testing for various components of aphasia
Naming
 The degree of anomia is useful as an overall index of the severity of a language deficit,
and is a prominent feature in virtually all post-stroke aphasic patients, in moderate stage
Alzheimer‘s disease, as well as semantic dementia.
 Naming ability requires an integration of visual, semantic, and phonological aspects of
item knowledge.
 There is a notable frequency effect, and rather than using very common items to test the
patient, such as a pen or watch, it may be more informative to ask about a winder, nib,
cufflinks, or a stethoscope.
 Phonemic paraphasias (for example, ‗‗baby flitter‘‘ for ‗‗baby sitter‘‘), and semantic
paraphasias (‗‗clock‘‘ for ‗‗watch‘‘, or ‗‗apple‘‘ for ‗‗orange‘‘) may also be seen, and
reflect pathology in Broca‘s area and the posterior perisylvian region, respectively.
 Broad superordinate responses, such as ‗‗animal‘‘, may be given in response to pictures
of, for example, a camel, with the progressive semantic memory impairment seen in
semantic dementia.
 Posterior lesions, particularly of the angular gyrus, can produce quite pronounced
anomia for visually recognized objects, and may be associated with alexia.
Comprehension
 It is useful to assess comprehension in a graded manner, starting with simple and then
more complex instructions. Use several common items (coin, key, pen), and ask the
patient to point each one in turn in order to assess single word comprehension.

Neurosciences Answers 133


© 2010 SPMM Course www.spmmpsychiatrycourse.com

 There is a frequency effect, and if this test seems too easy, try harder items around the
room.
 Sentence comprehension can be tested with several common items in order to devise
syntactically complex commands. For example, ‗‗touch the pen, and then the watch‘‘,
followed by more difficult sentences such as ‗‗touch the watch, after touching the keys
and the pen‘‘. Alternatively, ask ‗‗If the lion ate the tiger, who remained?‘‘.
 Syntactic ability is classically impaired with lesions of Broca‘s area or the anterior insular
region, and is commonly accompanied by phonological errors and poor repetition.
 Conceptual comprehension (i.e. understanding) can be assessed using the same
objects—for example, which of these items is used for recording the passage of time?
Similarly, one can ask which bird flies mainly at night and hoots. This type of naming to
definition helps exclude a visual deficit, while accessing the semantic store.
Repetition
 Use a series of words and sentences of increasing complexity. Repetition of
‗hippopotamus‘‘ followed by enquiry as to the nature of the animal assesses
phonological, articulator, and semantic processing simultaneously. Other useful words
are ‗‗aubergine‘‘, ‗‗emerald‘‘, and ‗‗perimeter‘‘.
 Sentence repetition can be tested with the well known phrase, ‗‗No ifs, ands or buts‘‘,
which is somewhat surprisingly more difficult than repeating ‗‗the orchestra played and
the audience applauded‘‘.
Letter and category fluency
 Patients are asked to produce as many words as possible starting with a particular letter of
the alphabet (F, A, and S are the commonly used letters). Proper names, and the
generation of exemplars from a single stem (for example, pot, pots, potter) are not
allowed.
 Category fluency is performed by, for example, asking for as many animals as possible in
one minute. Young adults can produce 20 animals, 15 animals is low average, and less
than 10 is definitely impaired.
 Letter fluency is usually more difficult (a score of 15 words per letter is normal), and
subjects with subcortical or frontal pathology score poorly on both measures, but worse
on letter fluency.
 In contrast, patients with semantic deficits, such as semantic dementia or Alzheimer‘s
disease, has a more pronounced impairment for categories. Refinements, such as
categories of dogs, can be introduced to detect more subtle deficits.

Reading
 Failure to comprehend is usually accompanied by an inability to read aloud, but the
reverse is not necessarily true. Test this either by writing a simple command ‗‗Close your
eyes‘‘ or using a few phrases from a nearby newspaper.
 Patients with so-called pure alexia exhibit the phenomenon of letter-by-letter
reading, with frequent errors in letter identification.
 Neglect dyslexia, seen in right hemisphere damage, is usually confined to the
initial part of a word and can take the form of omissions or substitutions (for
example, ‗‗land‘‘ for ‗‗island‘‘, and ‗‗fish‘‘ for ‗‗dish‘‘).
 Surface dyslexics have difficulty in reading words with irregular spelling (for
example, ‗‗suite‘‘, ‗‗cellist‘‘, ‗‗dough‘‘), which indicates a breakdown in the
linkage of words to their underlying semantic meanings and is one of the
hallmarks of semantic dementia.
 Deep dyslexics are unable to read plausible non-words (for example, ‗‗neg‘‘,
‗‗glem‘‘, ‗‗deak‘‘), and make semantic errors (‗‗canary‘‘ for ‗‗parrot‘‘).

Neurosciences Answers 134


© 2010 SPMM Course www.spmmpsychiatrycourse.com

 Alexia without agraphia (pure word blindness) almost always involves an infarct to the
left posterior cerebral artery affecting splenium of the corpus callosum and left visual
cortex. So the affected person, who is still able to see with right visual cortex, cannot
undertake lexical word processing making him unable to read. But he can still write
normally
Writing
 Writing is more vulnerable to disruption than reading, and involves coordination of both
central (spelling) and more peripheral (letter formation) components.
 Central dysgraphias affect both written and oral spelling. These syndromes are
analogous to those seen in the acquired dyslexias, and can be tested similarly.
 In general, intact oral spelling in the face of written spelling impairments suggests a
writing dyspraxia or neglect dysgraphia. The former results in effortful, and often
illegible, writing with frequent errors in the shape or orientation of letters. Copying is
also abnormal.
 Neglect dysgraphia results in misspelling of the initial part of words, and is frequently
associated with other non-dominant parietal lobe deficits of visuospatial ability and
perceptual function
 A mixed central and peripheral dysgraphia with spelling errors that tend to be
phonologically plausible is commonly seen in corticobasal degeneration (CBD).

ACALCULIA

 Acalculia refers to the inability to read, write, and comprehend numbers, and is not
exactly the same as an inability to perform arithmetical calculations (anarithmetrica).
 Although simple calculation is sufficient for most purposes, a full assessment of this skill
requires the patient to write numbers to dictation, copy numbers and read them aloud.
 The patient should also be asked to perform oral arithmetic, written calculation, and
finally be tested in ability to reason arithmetically (for example, ‗‗If one buys two items
costing £1.27, and one costing 70p how much change would be received from tendering a
£5 note‘‘).

APRAXIA:

♣ Damasio and Geschwind (1985) , defined apraxia as demonstrating varying combinations


of the following disturbances in order of progressive dysfunction:
o The failure to produce the correct movement in response to a verbal command,
o The failure to correctly imitate a movement performed by the examiner,
o The failure to perform a movement correctly in response to a seen object and
o The failure to handle an object correctly

♣ Although a number of categories, such as limb kinetic, ideomotor, and ideational, exist,
these labels are seldom useful in clinical practice.
♣ It is more helpful to describe the apraxia by region (orobuccal or limb), and to provide a
description of impaired performance, recording both spatial and sequencing errors on
several different types of task.
♣ Apraxia is of limited localizing ability, but the left parietal and frontal lobes appear to be
of greatest importance.
♣ Orobuccal apraxia is closely associated with lesions of the left inferior frontal lobe and
the insula, and commonly accompanies the aphasia caused by lesions of Broca‘s area.
♣ Progressive, isolated limb apraxia is virtually diagnostic of corticobasal degeneration.

Neurosciences Answers 135


© 2010 SPMM Course www.spmmpsychiatrycourse.com

♣ Oculomotor apraxia: Horizontal eye movement diffuclties seen in children. It is a rare


degenerative disorder of the nervous system that usually affects the ability of the patients
to quickly move their eyes from side to side and often have to turn their head (head
jerking) and not just their eyes to track an object

Types of apraxia:

Functional classification:
Apraxia type Definition Localization

Constructional Inability to construct elements to a Typically caused by a lesion


apraxia meaningful whole. E.g. inability to draw in the right cerebral
or copy simple diagrams or figures. hemisphere.
Ideational/conceptual Inability to perform a multiple-step task. most have damage to left
Patient does not know what to do. Errors hemisphere, typically involve
include, impairment in carrying out left parieto-occipital and
sequences of actions requiring the use of parietotemporal regions
various objects in the correct order so as
to achieve an intended purpose.
Ideomotor Disorder of goal-directed movement. mainly in left hemisphere;
Patient knows what to do but not how to frontal and parietal
do it. Impairment of pantomiming association areas. Unilateral
ability to use tool. Abnormalities include lesions of the left hemisphere
(i) abnormal amplitude; (ii) body-part- in right-handed patients
as-object substitution, e.g. the patient produce bilateral deficits,
uses his own finger to represent a usually less severe in the left
toothbrush when asked to brush his than in the right limb
teeth; (iii) abnormal orientation of body
part performing the action, and temporal most common type
errors. Improves on imitation and with
use of actual tool. Tool use more
affected than gestures.

Regional classification:
It is the inability to coordinate and carry Probably the most frequent types
Buccofacial out facial and lip movements such as of all apraxia due to focal brain
apraxia (aka whistling, winking, coughing, etc, on lesions. With rare exceptions,
facial-oral command. cannot perform skilled been associated with left
apraxia) movements involving the lips, mouth, and hemispheric lesions in right-
tongue in the absence of paresis handers.
Limb-kinetic Loss of hand and finger dexterity resulting Dominant Fronto- parietal or
from inability to connect or isolate primary motor cortex
individual movements. Affects use of
tools, gestures, especially distal fingers
movements.
Can be either ideomotor or ideational type
Other variants: Apraxia of speech, apraxia of eyelid
opening and apraxia of gait.

Zadikoff C and Lang AE. (2005) Apraxia in movement disorders. Brain 128:1480–97

Neurosciences Answers 136


© 2010 SPMM Course www.spmmpsychiatrycourse.com

EXECUTIVE FUNCTION
 This includes planning, initiation, sequencing, coordinating, error detection, error
correction, set shifting, and termination. It is closely allied to other frontal functions such
as judgement, problem solving, impulse control, and abstract reasoning.
 Executive function is generally believed to be a dorsolateral frontal lobe function and
depends on intact frontal-subcortical circuits.
 Impulsivity is thought to reflect failure of response inhibition, and is seen in inferior
frontal pathology. It can be assessed using the Go-No-Go task. The examiner instructs
the patient to tap once in response to a single tap, and to withhold a response for two taps.
This test can be made more difficult by changing the initial rule after several trials (for
example, ‗‗tap once when I tap twice, and not at all when I tap once‘‘).
 The ability to switch task, and the inhibition of inappropriate, or perseverative, responses
can also be assessed by asking the patient to copy a short sequence of alternating
squares and triangles, and then to continue across the page. Perseveration in drawing
one or other of the shapes may be seen in frontal lobe deficits, but the test is relatively
insensitive.
 The cognitive estimates test may prompt bizarre or improbable responses in patients
with frontal or executive dysfunction. Although it is a formal test performed at the
bedside by asking, for example, the height of the Post Office Tower, the population of
London, or the speed of a typical racehorse.
 Questions about the similarity between two conceptually similar objects can be used to
assess inferential reasoning which may be impaired in the same way. Simple pairs such
as ‗‗apples and oranges‘‘ or ‗‗desk and chair‘‘ are tested first, followed by more abstract
pairs such as ‗‗love and hate‘‘ or ‗‗sculpture and symphony‘‘. Patients typically answer,
quite concretely, that two objects are ‗‗different‘‘ or that they are ‗‗not similar‘‘ instead
of forming an abstract concept to link the pair. This often persists despite encouragement
to consider other ways in which the items are alike.
 Testing of proverb meanings probably measures a similar skill, but it is highly
dependent on pre-morbid educational ability and cultural background.

VISUOSPATIAL ABILITY
 Information from the visual cortex is directed towards the temporal or parietal cortex via
one of two streams.
 The dorsal (‗‗where‘‘) stream links visual information with spatial position and orientation
in the parietal lobe, whereas the ventral (‗‗what‘‘) stream links this information to the store
of semantic knowledge in the temporal lobes.
 The frontal eye fields are important in directing attention towards targets in the visual field.
 Visual neglect may produce a failure to groom one half of body, or eat what is placed on
one side of a plate.
Neglect
 Neglect of personal and extrapersonal space is usually caused by lesions
to the right hemisphere—usually the inferior parietal or prefrontal
regions.
 Left side of personal and extrapersonal space is represented only on right
parietal lobe; but right personal and extrapersonal space gets bilateral
representation. Hence a left sided lesion rarely results in neglect; but
right sided lesion can result in left sided neglect.
 Deficits can be uncovered by simultaneous bilateral sensory or visual
stimulation, or having the patient bisect lines of variable length.
 Letter and star cancellation tasks are similar, more formal tasks.

Neurosciences Answers 137


© 2010 SPMM Course www.spmmpsychiatrycourse.com

 Patients with object centered neglect fail to copy one side of an object,
and neglect dyslexics may not read the beginning of a line or word.
 Patients with anosognosia deny they are hemiplegic or even that the
affected limb belongs to them.
Dressing and constructional apraxia
 Although deficits in dressing and constructional ability are termed
apraxias, they are best considered as visuospatial, rather than motor
impairments.
 Copying three dimensional shapes such as a wire cube, interlocking
pentagons, or constructing a clock-face with numbers are good tests of
constructional ability, and may also highlight neglect if it is present.
 Left sided lesions tend to cause over-simplification in copying, whereas
right sided lesions may result in abnormal spatial relationships between
the constituent parts of the figure.
 Dressing apraxia is easily tested by having the patient put on clothing
that has been turned inside out.

AGNOSIA:
Visual agnosias
o Visual object agnosias cause a failure of object recognition despite adequate perception.
o Those with apperceptive visual agnosia have normal basic visual functions, but fail on
more complex tasks involving object identification and naming. However, they are able
to name objects to description, or by touch, indicating a preserved underlying semantic
representation of the object. This phenomenon is described with widespread, bilateral
occipitotemporal infarction.
o In cases of associative visual agnosia, the deficit reflects a disruption of stored semantic
knowledge, and involves all modalities accessing this information. Lesions of the anterior
left temporal lobe are typical.
o To test for these syndromes, it is necessary to assess object naming and description, along
with tactile naming, naming unseen objects to description, and the ability to provide
semantic information about unnamed items.
Prosopagnosia
 Prosopagnosics cannot recognise familiar faces. Often other clues, such as gait, voice
or distinctive clothing, are used to aid identification.
 The deficit may not be entirely selective to faces, and often fine grained identification
within categories may also be impaired (for example, makes of car, and types of
flowers).
 Patients are generally able to characterise individual facial features, and since the
underlying (semantic) knowledge associated with a particular person is not disrupted,
the ability to produce attributes of the face in question, if it is named, remains intact.
 Face processing is a bilateral function; more key areas may be present on right
hemisphere.
 Acquired prosopagnosia is usually associated with bilateral or right-sided lesions of the
occipital - temporal junction (FUSIFORM GYRUS). In rare cases of prosopagnosia
after left-sided lesions in left-handed subjects, it is attributed to a reversed hemispheric
specialization for face processing.
 An inferior occipitotemporal lesion, bordering on nondominant parietal lobe underlies
this disability, and is often associated with a field defect, achromatopsia or pure alexia.
Colour deficits

Neurosciences Answers 138


© 2010 SPMM Course www.spmmpsychiatrycourse.com

 Colour processing deficits such as achromatopsia (loss of ability to discriminate


colours) are often associated with pure alexia after medial occipitotemporal damage,
following left posterior cerebral artery infarction.
 Colour agnosia impairs tasks requiring retrieval of colour information (for example
‗‗what colour is a banana?‘‘)
 Colour anomia (for example ‗‗What colour is this?‘‘) refers to a specific disorder of
colour naming despite intact perception and colour knowledge, probably caused by a
disconnection of the language structures in the temporal lobe from the visual cortex.

A few rare syndromes are worthy of mention.


Balint’s syndrome consists of a triad of
 simultanagnosia (inability to attend to more than one item of a complex scene at a time),
 optic ataxia (inability to guide reaching or pointing despite adequate vision), and
 oculomotor apraxia (inability to voluntarily direct saccades to a visual target).
Fields may be full when challenged with gross stimuli, and oculocephalic reflexes are intact. This
syndrome results from bilateral damage including the superior-parieto-occipital region, which
disrupts the dorsal (‗‗where‘‘) visual processing stream linking visual with parietal association
areas. Possible causes include carbon monoxide poisoning, watershed infarction, leucodystrophy,
and the posterior cortical variant of Alzheimer‘s disease.

Gerstmann syndrome is characterized by four primary symptoms:


 Dysgraphia/agraphia
 Dyscalculia/acalculia
 Finger agnosia
 Left-right disorientation
This is rarely seen as full presentation of tetrad; It is often associated with brain lesions in the
dominant (usually left) side of the angular and supramarginal gyri (parietal lobe).

Anton’s syndrome is a visual agnosia, in which the patient denies any deficit and may attempt to
negotiate the environment, invariably without success.
In the curious phenomenon known as blindsight, visual stimuli can induce a response despite
cortical blindness. It is probably mediated by perceptual processing in subcortical structures and
brainstem nuclei.

Marchiafava-Bignami disease is due to demyelination and necrosis of corpus callosum and


adjacent anterior commissure seen in alcoholics, especially using red wine excessively (not clear
whether some impurities are implicated). Patients present with sudden onset of stupor or coma
and seizures. A chronic onset of dementia and/or gait problems with spasticity is also reported.
2 clinical types are identified: Type A had predominant features of coma and stupor. This
subtype is associated with a high prevalence of pyramidal-tract symptoms. Radiologic features
include involvement of the entire corpus callosum. Type B is characterized by normal or mildly
impaired mental status. Radiologic features are partial or focal callosal lesions.

Barton, JJS. Prosopagnosia associated with a left occipitotemporal lesion. Neuropsychologia. 2008
46(8):2214-24

J. Neurol. Neurosurg. Psychiatry 2005;76;22-30

Cranial nerves:

Neurosciences Answers 139


© 2010 SPMM Course www.spmmpsychiatrycourse.com

No. Name Main clinical action


I Olfactory Smell Pure sensory
II Optic Vision, fields, afferent light reflex Pure sensory
III Oculomotor Eyelid elevation, eye elevation, Pure motor
ADduction, depression in ABduction,
efferent (pupil)
IV Trochlear Eye intorsion, depression in Pure motor
ADduction
V Trigeminal Facial and corneal sensation, muscles Sensory and motor
of mastication
VI Abducens Eye ABduction Pure motor
VII Facial Facial movement, taste fibres Sensory and motor
VIII Vestibular Balance Sensory
Cochlear Hearing Sensory
IX Glossopharyngeal Sensation - soft palate, taste fibres Sensory and motor
X Vagus Cough, palatal and vocal cord Sensory and motor
movements
XI Accessory Head turning, shoulder shrugging Pure motor
XII Hypoglossal Tongue movement Pure motor
Adapted from Kumar & Clark Textbook of clinical medicine 6 th edition Pg 1179

Olfactory nerve CN I:
 Only sensory nerve to have no thalamic relay;
 Unilateral anosmia should raise the suspicion of a lesion affecting the olfactory nerve
filaments, bulb, tract, or stria.
 Because the cortical representation for smell in the piriform cortex is bilateral, a
unilateral lesion distal to the decussation of the olfactory fibers (i.e. temporal/ uncinate)
causes no olfactory impairment.
 Frontal meningiomas can cause unilateral anosmia.
 Head injury is probably the most common cause of disruption of the olfactory fibers
Hyposmia is an early feature of Parkinson‘s disease and Alzheimer‘s dementia and may
precede motor and cognitive signs respectively.
 Impaired sense of smell is seen in some patients at 50% risk of Parkinsonism.

Optic nerve CN II:

Syndrome Lesion
Unilateral one eye Lesion anterior to optic chiasm e.g. optic nerve itself or
blindness retina
Bitemporal Optic chiasmatic leasion e.g. cranipharyngioma, pineal
hemianopia tumors
Homonymous lesions of the right sided optic tract, lateral geniculate
hemianopia – left body, optic radiations, and striate cortex (any retro
chiasmatic structure)
Homonymous lesions of the left retro chiasmatic structures
hemianopia – right
Enlargement of the any process causing disc swelling
blind spot
Superior Optic irradiation lesion at temporal lobes of contra lateral

Neurosciences Answers 140


© 2010 SPMM Course www.spmmpsychiatrycourse.com

quandrantonopia side
inferior Optic irradiation lesion at parietal lobes of contra lateral
quandrantonopia side
Cortical blindness Occipital cortex lesions

o Hemianopia is a field defect that encompasses roughly half of the field. Vertical hemianopia
can be nasal or temporal. Horizontal or altitudinal hemianopia can be superior or inferior.
o When only one-fourth of the field is affected, the resulting deficit is called quadrantanopia.
o Bilateral field defects are said to be homonymous when they are similarly located in both
visual fields
o Funnel vision: With an organic field defect, the field projected at 2 m is larger than the field
plotted at 1 m (funnel vision). Seen in glaucoma, retinitis pigmentosa, CAR, hyaline bodies
of the disc, postpapilledema optic atrophy, bilateral occipital infarcts with macular sparing,
and feigned visual loss.
o Tunnel vision refers to absence of such projected disparity and having patchy spirals of field
loss – seen in hysteria or malingering.
o Cortical blindness is most often due to simultaneous or successive posterior cerebral artery
occlusion. In effect it is a bilateral homonymous hemianopia. These patients may be left with
a small central field around the point of fixation (macular sparing or keyhole vision) or may
have complete blindness.
o Occasionally, patients with cortical blindness deny their visual defect (Anton's syndrome).

The following testing is appropriate for optic nerve:


 Acuity using the Snellen chart (near and distant vision)
 Visual fields using confrontation test or perimetry
 Colour vision using Ishihara chart
 Fundoscopy

Pupillary light reflex.


Afferent fibres in each optic nerve (some crossing in the chiasm)
pass to both lateral geniculate bodies and relay to the Edinger-
Westphal nuclei (midbrain) via the pretectal nucleus.
Efferent (parasympathetic) fibres from each Edinger-Westphal
nucleus pass via the third nerve to the ciliary ganglion and thence to
the pupil.
Light constricts the pupil being illuminated (direct reflex) and, by the
consensual reflex, the contralateral pupil.

The convergence / accommodation reflex


Fixation on a near object requires convergence and is accompanied
by pupillary constriction. Afferent fibres in each optic nerve, which
pass through both lateral geniculate bodies, also relay to the
convergence centre. This centre receives muscle spindle afferent
fibres from the extraocular muscles - principally medial recti - which
are innervated by the third nerve.
The efferent route is from the convergence centre to the Edinger-
Westphal nucleus, ciliary ganglion and pupils.

o Pupils that accommodate but do not react are said to show light-near dissociation.
 Two important types are Argyll Robertson pupil, seen in neurosyphilis and
diabetes (more common these days), and Adie pupil due to peripheral

Neurosciences Answers 141


© 2010 SPMM Course www.spmmpsychiatrycourse.com

pupillary defect producing a tonic pupil. ARP (note: Accommodation Reflex


Present –light reflex absent) is due to afferent defect in pupillary reflex
pathway – possibly pretectal.

Oculomotor nerve - CN III


► The oculomotor nucleus of the nerve is located in the midbrain
► Supplies the levator palpebrae superioris; the superior, inferior, and medial recti; and the
inferior oblique muscles.
► Lesions of CN III result in paralysis of the ipsilateral upper eyelid and pupil, leaving the
patient unable to adduct and look up or down. The eye is frequently turned out (exotropia).
► Lesions at the nucleus of the third nerve cause bilateral ptosis, in addition to the findings
mentioned above.
► Paralysis of CN III is the only ocular motor nerve lesion that results in diplopia in more
than 1 direction.
► Pupillary involvement is an additional clue to involvement of CN III.
► Pupil-sparing CN III paralysis occurs in diabetes mellitus, vasculitides of various
etiologies, and certain brainstem lesions such as due to multiple sclerosis.
Trochlear nerve - CN IV
The nucleus of the nerve is located in the midbrain. It innervates the superior oblique muscle.
Trochlear nerve typically allows a person to view the tip of his or her nose.
Trigeminal nerve - CN V
 The nucleus of the nerve stretches from the midbrain (ie, mesencephalic nerve)
through the pons (ie, main sensory nucleus and motor nucleus) to the cervical region
(ie, spinal tract of the trigeminal nerve).
 It provides sensory innervation for the face and supplies the muscles of mastication.
 Divisions: ophthalmic; V1, maxillary; V2, mandibular; V3.
 Corneal reflex:
 Afferent – V nerve
 Efferent – facial nerve
 Complete paralysis of CN V results in sensory loss over the ipsilateral face and
weakness of the muscles of mastication. Attempted opening of the mouth results in
deviation of the jaw to the paralyzed side.
 Acoustic neuroma can press on 5th nerve leading to loss of corneal reflex.
Abducens nerve - CN VI
The nucleus of the nerve is located in the paramedian pontine region in the floor of the fourth
ventricle. It innervates the lateral rectus, which abducts the eye. Patients complain of double
vision on horizontal gaze only. This finding is referred to as horizontal homonymous diplopia.
Paralysis of CN VI is a false localising sign as it may result from increased intra cranial
pressure.
Facial nerve - CN VII
 Motor supply to facial muscles from motor nucleus.
 Though it is considered a predominantly motor nerve, it also innervates a small strip
of skin of the posteromedial aspect of the pinna and around the external auditory
canal. It serveds to conduct taste sensation from the anterior two thirds of the tongue
and relay to sensory nucleus tractus solitarius.
 Secrotomotor functions inlue parasympathetic relay to lacrimal, lingual and
submandibular glands.
 A lower-motor-neuron lesion of the nerve, results in complete ipsilateral facial
paralysis; the face draws to the opposite side as the patient smiles. Eye closure is
impaired, and the ipsilateral palpebral fissure is wider. This is called Bell ’s palsy
where cause is idiopathic.

Neurosciences Answers 142


© 2010 SPMM Course www.spmmpsychiatrycourse.com

 In an upper motor neuron lesion, only the lower half of the face is paralyzed. Eye
closure is usually preserved.
Vestibulocochlear nerve - CN VIII
► 2 components – vestibular for nbalance; cochlear for hearing.,
► Auditory part tested using 512 Hz – Weber‘s test and Rinne‘s test.
► The Weber test involves holding a vibrating tuning fork against the forehead in the
midline. The vibrations are normally perceived equally in both ears because bone
conduction is equal. In conductive hearing loss, the sound is louder in the abnormal
ear than in the normal ear. In sensorineural hearing loss, lateralization occurs to the
normal ear.
► In the Rinne test, the vibrating tuning fork is placed over the mastoid region until
the sound is no longer heard. It is then held at the opening of the ear canal on the
same side. A patient with normal hearing should continue to hear the sound. In
conductive hearing loss, the patient does not continue to hear the sound, since bone
conduction in that case is better than air conduction. In sensorineural hearing loss,
both air conduction and bone conduction are decreased to a similar extent.
► The vestibular portion transmits information about linear and angular accelerations
of the head from the utricle, saccule, and semicircular canals of the membranous
labyrinth to the vestibular nucleus.
► The Romberg test is performed to evaluate vestibular control of balance and
movement. When standing with feet placed together and eyes closed, the patient
tends to fall toward the side of vestibular hypofunction. Results of the Romberg test
may also be positive in patients with polyneuropathies, and diseases of the dorsal
columns, but these individuals do not fall consistently to 1 side as do patients with
vestibular dysfunction.
► Provocative tests include caloric testing. Normally on cold water testing, nystagmus
is noted to opposite side; warm water elicits nystagmus towards same side. (COWS
mnemonic)
Glossopharyngeal nerve - CN IX
 The nucleus of the CN IX is anatomically indistinguishable from the CN X - called
nucleus ambiguous. Its main function is sensory innervation of the posterior third of the
tongue and the pharynx. It also innervates the pharyngeal musculature, particularly the
stylopharyngeus, in concert with the vagus nerve.
 Vascular stretch afferents from the aortic arch and carotid sinus travel via
glossopharyngeal nerve to the nucleus solitarius – important for neural control of blood
pressure.
 Lesions affecting the glossopharyngeal nerve result in loss of taste in the posterior third
of the tongue and loss of pain and touch sensations in the same area, soft palate, and
pharyngeal walls.
 CN IX and CN X travel together, and their clinical testing is not entirely separable.
Vagus nerve - CN X
 Starting in the nucleus ambiguous, the vagus nerve has the longest peripheral course
of all cranial nerves – it stretches upto splenic flexure of colon.
 Provides motor supply to the pharyngeal muscles (except the stylopharyngeus and the
tensor veli palati), palatoglossus, and larynx.
 It innervates the smooth muscles of the tracheobronchial tree, esophagus, and GI tract
up to the junction between the middle and distal third of the transverse colon.
 Somatic sensation is carried from the back of the ear, the external auditory canal, and
parts of the tympanic membrane, pharynx, larynx, and the dura of the posterior fossa.

Neurosciences Answers 143


© 2010 SPMM Course www.spmmpsychiatrycourse.com

 The pharyngeal gag reflex (ie, tongue retraction and elevation and constriction of the
pharyngeal musculature in response to touching the posterior wall of the pharynx,
tonsillar area, or base of the tongue) and the palatal reflex (ie, elevation of the soft
palate and ipsilateral deviation of the uvula on stimulation of the soft palate) are
decreased in paralysis of CN IX and CN X.
 In unilateral CN IX and CN X paralysis, touching these areas results in deviation of
the uvula to the normal side.
Spinal accessory nerve - CN XI
Spinal root supplies trapezius and cternocleidomastoid.
Hypoglossal nerve - CN XII
It provides motor innervation for all the extrinsic and intrinsic muscles of the tongue. To test the
hypoglossal nerve, have the patient protrude the tongue; when paralyzed on 1 side, the tongue
deviates to the side of paralysis on protrusion.

Neurological signs:

Absent ankle jerks, upgoing plantars:


Odd combination - UMN lesion of corticospinal tracts is expected to cause exaggerated ankle
reflex (i.e. clonus) with upgoing plantar normally. But in subacute combined degeneration cord,
Syphilitic taboparesis and Freidreich‘s ataxia and MND we see absence of ankle jerk as spinal
reflex pathway is affected (afferent) while UMN type damage still produces Babinski – upgoing
plantar.

Anisocoria
This refers to pupillary asymmetry, which may result from sympathetic or parasympathetic
dysfunction. Sympathetic dysfunction results in Horner syndrome, in which the pupil is small but
reacts to light. Parasympathetic dysfunction results in tonic pupil.

Argyll-Robertson pupil, seen in neurosyphilis, is irregular and small; it does not react to light,
but does accommodate.

Anosognosia
This refers to denial of illness and typically is seen in patients with right frontoparietal lesions,
resulting in left hemiplegia that the patient denies. A form of visual anosognosia (Anton
syndrome) is seen in patients with bilateral occipital lobe infarctions; these patients with double
hemianopsia (bilateral cortical blindness) deny that they are blind.

Asterixis
. Momentary loss of tone and flapping of the hand are seen when the patient extends his arms in
front with the wrists dorsiflexed. This is seen in patients with metabolic encephalopathies

Beevor sign
This is seen with bilateral lower abdominal paralysis that results in upward deviation of the
umbilicus when the patient tries to raise his head and sit up from the supine, recumbent position.

Brown Sequard syndrome:


This is due to hemisection of spinal cord; full syndrome is rare. Clinical features include
 Interruption of the lateral corticospinal tracts
o Ipsilateral spastic paralysis below the level of the lesion
o Babinski sign ipsilateral to lesion
o Abnormal reflexes (UMN type – hyperreflexia)

Neurosciences Answers 144


© 2010 SPMM Course www.spmmpsychiatrycourse.com

 Interruption of posterior white column –


o Ipsilateral loss of tactile discrimination, vibratory, and position sensation below
the level of the lesion
 Interruption of lateral spinothalamic tracts:
o Contralateral loss of pain and temperature sensation. This usually occurs 2-3
segments below the level of the lesion.
Chvostek sign
This is seen in hypocalcemia. Tapping the cheek at the angle of the jaw precipitates tetanic facial
contractions.

Doll's-eye maneuver
This refers to turning the head passively with the patient awake and fixated or when the patient is
in a coma. In the former, the eyes remain fixated at the original focus when all gaze pathways are
normal; in the latter, the eyes deviate in the opposite direction when the brainstem is intact.

Friedreich’s ataxia is an inherited neurological disease (trinucleotide repeat) with pes cavus,
kyphoscoliosis, cerebellar signs, impaired joint position / vibration, cardiomyopathy, optic
atrophy.

Gower sign
This sign, seen in severe myopathies, occurs when the patient attempts to stand up from the floor.
Patients first sit up, then assume a quadrupedic position, and then climb up their own legs by
using their arms to push themselves up.

Holmes-Adie syndrome - A benign form of tonic pupil is seen in Holmes-Adie syndrome i.e.,
tonic pupil with absent patellar and Achilles reflexes.

Horner's syndrome
Remember PAMELA – Ptosis, Anhidrosis, Miosis, Enophtholmos and Loss of ciliospinAl reflex.
This collection of signs indicates a lesion of the sympathetic pathway on the same side. Seen in
cervical lesions –e.g. apical lung tumour affecting cervical sympathetic ganglion, carotid
aneurysms.

Kayser-Fleischer ring
This is a brownish ring around the limbus of the cornea. It is best demonstrated during an
ophthalmologic slitlamp examination.

Marcus-Gunn pupil
This sign requires a swinging-flashlight test to assess. As the flashlight swings from 1 eye to the
other, the abnormal pupil dilates as the light swings back from the normal side. No anisocoria is
seen. The phenomenon is also called a paradoxical pupillary reflex and indicates an afferent
(optic nerve) pupillary defect.

Mononeuritis multiplex:
Painful asymmetric asynchronous sensory and motor peripheral neuropathy with isolated damage
to at least 2 separate nerve areas.
Causes: diabetes, vasculitis, amyloidosis, direct tumor involvement, autoimmune disorders
paraneoplastic syndromes.

Milkmaid's grip
This refers to the inability to maintain a sustained grip commonly seen in patients with chorea.

Neurosciences Answers 145


© 2010 SPMM Course www.spmmpsychiatrycourse.com

Myerson sign
Patients with Parkinson disease, particularly those with bilateral frontal lobe dysfunction,
continue to blink with repeated glabellar taps.

Optic neuritis:
The classic triad of optic neuritis consists of (1) loss of vision, (2) eye pain, and (3)
dyschromatopsia. 70% unilateral. Usually recover spontaneously (Multiple sclerosis) within 2-3
weeks. Movement- or sound-induced phosphenes seen. Reduction in vision may worsen in bright
light, a symptom that seems paradoxical. The Uhthoff symptom, described as exercise- or heat-
induced vision loss is seen in 50% of patients. Afferent pupillary defect noted on testing (i.e.
direct light reflex absent; consensual present)

Subacute combined degeneration is due to vitamin B12 deficiency; causes peripheral


neuropathy, posterior column signs with pyramidal signs below waist.

Trombone tongue
This is seen in patients with chorea. It refers to the unsteadiness of the tongue when the patient
tries to protrude it outside the mouth.

http://www.emedicine.com/neuro/TOPIC632.HTM#section~References

UMN vs. LMN lesions


• Upper motor neuron • Lower motor neuron
• Rigidity • Atonia or hypotonia
• Hypertonia • Loss of Deep tendon
• Exaggerated reflexes reflexes
• Mild atrophy only – • Atrophic, wasted
disuse • Fasciculations
• E.g. pseudobulbar • E.g. bulbar palsy
palsy

Neurosciences Answers 146


© 2010 SPMM Course www.spmmpsychiatrycourse.com

Bulbar palsy Pseudobulbar palsy


• LMN weakness of 9-12 • Bilateral supranuclear
cranial nerves (UMN) lesions of lower
• Wasted, fasciculating cranial nerves
tongue • Stiff tongue; Wasting seen
• Nasal speech only in later stages
• Lost jaw jerk and gag • Donald duck speech
reflex • Exaggerated jaw jerk;
• Emotional lability not seen preserved gag reflex
• poliomyelitis, myasthenia • Emotional lability
gravis, botulism, or (pathological emotionalism)
muscular dystrophies. seen due to cortical damage
• Motor neuron disease
(casues both
bulbar/pseudobulbar),
multiple sclerosis,
multiinfarct dementia and
sever head injury.

Traumatic brain injury:


Peak incidence of head injury is between the ages of 15 - 24 years

 Traumatic brain injury is the result of mechanical forces applied to the skull and
transmitted to the brain. This may lead to focal and/or diffuse brain damage.
 Focal lesions often result from a direct blow to the head and include brain laceration,
contusion, intracerebral hemorrhage, subarachnoid or subdural hemorrhage, and ischemic
infarct.
 Concussion damage causes transient coma for hours followed by apparent complete
clinical recovery. Brain contusion leads to prolonged coma, focal signs and lasting brain
damage. Pathological support for this division is poor.
 Contusion occurs directly beneath (coup injury) or contralateral (contre-coup injury) to
the site of impact. Contre-coup is most common in the orbital–frontal area and the
temporal tips, where acceleration/deceleration forces cause the brain to impact on the
bony protuberances of the skull. A frontal behavioural dyscontrol syndrome as described
in the question occurs due to bilateral orbito-frontal injury.
 Mechanisms of TBI:
 axonal and neuronal damage - shearing and rotational stresses on decelerating
brain, often at sites opposite impact (contre-coup effect)
 axonal and neuronal damage from direct trauma
 brain oedema and raised intracranial pressure

Neurosciences Answers 147


© 2010 SPMM Course www.spmmpsychiatrycourse.com

 brain hypoxia
 brain ischaemia.
 Differential motion of the brain within the skull can cause shearing and stretching of the
axons resulting in diffuse axonal injury (DAI). In diffuse axonal injury (DAI) damage
occurs over a more widespread area than in focal brain injury with extensive lesions in
white matter tracts. This causes unconsciousness and persistent vegetative state with the
outcome frequently being coma. A high number of patients with severe DAI never regain
consciousness.
 Two types of amnesia can occur after head injury:
 Post-traumatic amnesia (PTA) includes amnesia for the period of
injury and the period following injury until normal memory resumes.
This is anterograde.
 Retrograde amnesia includes dense amnesia for the period between the
last clearly recalled memory prior to the injury and the injury itself.
Generally in minutes, it reduces with time gradually.
 Poor prognostic factors with respect to psychiatric morbidity following head injury
includes long duration of loss of consciousness, long PTA, elderly, chronic alcohol use,
diffuse brain damage, new onset seizures and focal damage to dominant lobe.
 GCS at 24 hours after injury is widely used to assess severity.
 Apart from GCS (Glasgow coma scale) alternative indices of TBI severity such as length
of coma (LOC), duration of post-traumatic amnesia (PTA), and the Abbreviated Injury
Scale (AIS) have also been used to determine injury severity and establish prognosis for
recovery
 LOC and PTA have been used exclusively to predict functional outcome, but the AIS
has been used to predict survival
 Most investigations have found LOC or PTA to be more predictive of functional status
than GCS.
Duration of PTA Classification Functional outcome

PTA less than 60 mild injury May return to work in <1


minutes month
PTA between 1-24 hours moderate injury May return to work in 2
months
PTA between 1-7 days severe injury May return to work in 4months

PTA greater than 7 days very severe injury May require > 1 year for return
to work

 Late sequelae:
o Cognitive impairment is common especially after closed head injuries with PTA
lasting >24 hours.
o Personality changes are most likely after head injury to the orbito-frontal lobe or
anterior temporal lobe.
o Depression (most common sequelae) and anxiety occur in roughly 1/4 of head
injury survivors. Suicide risk is also higher post head injury.

Neurosciences Answers 148


© 2010 SPMM Course www.spmmpsychiatrycourse.com

o Post-concussional syndrome is characterised by headache; dizziness; insomnia;


irritability; emotional lability; increased sensitivity to noise, light, etc; fatigue; poor
concentration; anxiety; and depression.
o A schizophrenia-like psychosis with prominent paranoia is associated with left
temporal injury, while affective psychoses (esp. mania in 9% patients) are
associated with right temporal or orbito-frontal injury.
o There is also an increased prevalence of schizophrenia post head injury (-2.5%
develop the disorder).
o Post-traumatic epilepsy is seen in 5% closed and 30% open head injuries (usually
during first year) and worsens prognosis.
o Less psychopathology in children after head injury due to increased brain plasticity.
Katz DI, Alexander MP. Traumatic brain injury: predicting course of recovery and outcome for
patients admitted to rehabilitation. Arch Neurol 1994; 51: 661–70

Further reading:
Most receptor related stuff and neurochemistry can be read from Kaplan & Sadock synopsis of
psychiatry.
Hodges Cognitive assessment for clinicians is a very good book for neuropsychology and
behavioural neurology.
Emedicine (www.emedicine.com) is a useful resource for clinical neurology topics.
Companion has a chapter on sleep disorders – quite useful.

Disclaimer:
These lecture notes are prepared by consulting various published sources including peer reviewed journals
and books. These are acknowledged wherever possible; due to the structure of this revision notes,
acknowledgements have not been possible for every passage/fact. We do not check the accuracy of drug
related information using external sources; no part of these notes should be used as prescribing information.

Lena Palaniyappan
May 2010
(Previous version: Lena Palaniyappan & Sree Murthy)

Neurosciences Answers 149


© 2010 SPMM Course www.spmmpsychiatrycourse.com

ANSWERS
1A 43A 85D EMI 1: 1E 2C 3D
2B 44C 86E 4A 5G 6F 7J 8H
9I
3A 45C 87A
4E 46E 88C EMI 2: 1C 2A
5D 47A 89C EMI 3: 1BC 2E
6D 48C 90A 3H
7C 49E 91E EMI 4: 1B 2I 3F
8D 50C 92B 4H 5J
9B 51B 93D EMI 5: 1L 2H
10E 52A 94A 3J 4H 5K
11B 53A 95E 6F 7G 8A 9I
12B 54C 96D EMI 6: 1F 2A
13C 55A 97A 3D 4I 5K 6G 7E
14C 56B 98B 8A
15A 57E 99D EMI 7: 1B 2C
16D 58E 100A 3AB 4F 5G
17D 59D 101A 6D 7E
18C 60D 102D EMI 8: 1BIJ 2C
19B 61B 103D 3G 4H 5D
20A 62B 104B EMI 9: 1AB
21C 63A 105C 2G 3H
22A 64E 106A EMI 10: 1F 2H
23E 65C 107B 3 A 4 B 5C 6I
24B 66D 108C 7E 8J 9G 10D
25C 67B 109C EMI 11: 1G 2I
26B 68E 110A 3H 4D 5A 6F
27E 69C 111E 7E 8C
28D 70B 112D EMI 12: 1J 2K
29B 71C 113D 3C 4B 5D
30D 72B 114C 6H
31D 73B 115A EMI 13: 1D 2C
32D 74A 116B 3F 4E 5B 6J
33B 75D 117A
34A 76B 118D EMI14: 1AD 2EJ
35C 77D 119B 3HI
36D 78C
37B 79C
38B 80D
39A 81C
40E 82A
Neurosciences Answers 41D 83D 150
42A 84D

Vous aimerez peut-être aussi